If visible light has more energy than microwaves, why isn't visible light dangerous?












43














Light waves are a type of electromagnetic wave and they fall between 600-700 nm long. Microwaves are less energetic but seem to be more dangerous than visible light. Is visible light dangerous at all and why not?










share|cite|improve this question




















  • 50




    how are microwaves dangerous?
    – ZeroTheHero
    2 days ago






  • 4




    @ZeroTheHero : depends on the intensity of microwave radiation. If it can cook our meals, it can cook us :-(. For example, it can cause blindness and sterility through thermal effect. See osha.gov/SLTC/radiofrequencyradiation/hazards.html
    – akhmeteli
    2 days ago






  • 18




    @akhmeteli The same amount of energy delivered through visible light will do comparable or even more significant damage (the only thing that would make visible light less dangerous would be part of the energy being reflected). As will falling into a pot of boiling water. That doesn't make microwaves "more dangerous than visible light". Of couse there are thermal effects, but those don't care about the wavelength besides determining how much of energy is absorbed.
    – Luaan
    yesterday






  • 26




    I'm pretty sure that if you stick your hand in a box with a 900W visible light generator, you get at least as serious damage as if you put your hand in a 900W microwave oven
    – Suppen
    yesterday






  • 18




    This is an odd question. Let me pose a question to you: A lead bullet is much more dense than a grand piano, and yet if you drop a grand piano on me from ten meters up, it will be much more harmful than if you drop a lead bullet on me from ten meters up. So are pianos more dangerous than bullets despite their low density? ("Should we have better piano control?" is a political question of course.)
    – Eric Lippert
    yesterday


















43














Light waves are a type of electromagnetic wave and they fall between 600-700 nm long. Microwaves are less energetic but seem to be more dangerous than visible light. Is visible light dangerous at all and why not?










share|cite|improve this question




















  • 50




    how are microwaves dangerous?
    – ZeroTheHero
    2 days ago






  • 4




    @ZeroTheHero : depends on the intensity of microwave radiation. If it can cook our meals, it can cook us :-(. For example, it can cause blindness and sterility through thermal effect. See osha.gov/SLTC/radiofrequencyradiation/hazards.html
    – akhmeteli
    2 days ago






  • 18




    @akhmeteli The same amount of energy delivered through visible light will do comparable or even more significant damage (the only thing that would make visible light less dangerous would be part of the energy being reflected). As will falling into a pot of boiling water. That doesn't make microwaves "more dangerous than visible light". Of couse there are thermal effects, but those don't care about the wavelength besides determining how much of energy is absorbed.
    – Luaan
    yesterday






  • 26




    I'm pretty sure that if you stick your hand in a box with a 900W visible light generator, you get at least as serious damage as if you put your hand in a 900W microwave oven
    – Suppen
    yesterday






  • 18




    This is an odd question. Let me pose a question to you: A lead bullet is much more dense than a grand piano, and yet if you drop a grand piano on me from ten meters up, it will be much more harmful than if you drop a lead bullet on me from ten meters up. So are pianos more dangerous than bullets despite their low density? ("Should we have better piano control?" is a political question of course.)
    – Eric Lippert
    yesterday
















43












43








43


7





Light waves are a type of electromagnetic wave and they fall between 600-700 nm long. Microwaves are less energetic but seem to be more dangerous than visible light. Is visible light dangerous at all and why not?










share|cite|improve this question















Light waves are a type of electromagnetic wave and they fall between 600-700 nm long. Microwaves are less energetic but seem to be more dangerous than visible light. Is visible light dangerous at all and why not?







energy visible-light electromagnetic-radiation estimation microwaves






share|cite|improve this question















share|cite|improve this question













share|cite|improve this question




share|cite|improve this question








edited yesterday









Qmechanic

102k121831160




102k121831160










asked 2 days ago









susesuse

362137




362137








  • 50




    how are microwaves dangerous?
    – ZeroTheHero
    2 days ago






  • 4




    @ZeroTheHero : depends on the intensity of microwave radiation. If it can cook our meals, it can cook us :-(. For example, it can cause blindness and sterility through thermal effect. See osha.gov/SLTC/radiofrequencyradiation/hazards.html
    – akhmeteli
    2 days ago






  • 18




    @akhmeteli The same amount of energy delivered through visible light will do comparable or even more significant damage (the only thing that would make visible light less dangerous would be part of the energy being reflected). As will falling into a pot of boiling water. That doesn't make microwaves "more dangerous than visible light". Of couse there are thermal effects, but those don't care about the wavelength besides determining how much of energy is absorbed.
    – Luaan
    yesterday






  • 26




    I'm pretty sure that if you stick your hand in a box with a 900W visible light generator, you get at least as serious damage as if you put your hand in a 900W microwave oven
    – Suppen
    yesterday






  • 18




    This is an odd question. Let me pose a question to you: A lead bullet is much more dense than a grand piano, and yet if you drop a grand piano on me from ten meters up, it will be much more harmful than if you drop a lead bullet on me from ten meters up. So are pianos more dangerous than bullets despite their low density? ("Should we have better piano control?" is a political question of course.)
    – Eric Lippert
    yesterday
















  • 50




    how are microwaves dangerous?
    – ZeroTheHero
    2 days ago






  • 4




    @ZeroTheHero : depends on the intensity of microwave radiation. If it can cook our meals, it can cook us :-(. For example, it can cause blindness and sterility through thermal effect. See osha.gov/SLTC/radiofrequencyradiation/hazards.html
    – akhmeteli
    2 days ago






  • 18




    @akhmeteli The same amount of energy delivered through visible light will do comparable or even more significant damage (the only thing that would make visible light less dangerous would be part of the energy being reflected). As will falling into a pot of boiling water. That doesn't make microwaves "more dangerous than visible light". Of couse there are thermal effects, but those don't care about the wavelength besides determining how much of energy is absorbed.
    – Luaan
    yesterday






  • 26




    I'm pretty sure that if you stick your hand in a box with a 900W visible light generator, you get at least as serious damage as if you put your hand in a 900W microwave oven
    – Suppen
    yesterday






  • 18




    This is an odd question. Let me pose a question to you: A lead bullet is much more dense than a grand piano, and yet if you drop a grand piano on me from ten meters up, it will be much more harmful than if you drop a lead bullet on me from ten meters up. So are pianos more dangerous than bullets despite their low density? ("Should we have better piano control?" is a political question of course.)
    – Eric Lippert
    yesterday










50




50




how are microwaves dangerous?
– ZeroTheHero
2 days ago




how are microwaves dangerous?
– ZeroTheHero
2 days ago




4




4




@ZeroTheHero : depends on the intensity of microwave radiation. If it can cook our meals, it can cook us :-(. For example, it can cause blindness and sterility through thermal effect. See osha.gov/SLTC/radiofrequencyradiation/hazards.html
– akhmeteli
2 days ago




@ZeroTheHero : depends on the intensity of microwave radiation. If it can cook our meals, it can cook us :-(. For example, it can cause blindness and sterility through thermal effect. See osha.gov/SLTC/radiofrequencyradiation/hazards.html
– akhmeteli
2 days ago




18




18




@akhmeteli The same amount of energy delivered through visible light will do comparable or even more significant damage (the only thing that would make visible light less dangerous would be part of the energy being reflected). As will falling into a pot of boiling water. That doesn't make microwaves "more dangerous than visible light". Of couse there are thermal effects, but those don't care about the wavelength besides determining how much of energy is absorbed.
– Luaan
yesterday




@akhmeteli The same amount of energy delivered through visible light will do comparable or even more significant damage (the only thing that would make visible light less dangerous would be part of the energy being reflected). As will falling into a pot of boiling water. That doesn't make microwaves "more dangerous than visible light". Of couse there are thermal effects, but those don't care about the wavelength besides determining how much of energy is absorbed.
– Luaan
yesterday




26




26




I'm pretty sure that if you stick your hand in a box with a 900W visible light generator, you get at least as serious damage as if you put your hand in a 900W microwave oven
– Suppen
yesterday




I'm pretty sure that if you stick your hand in a box with a 900W visible light generator, you get at least as serious damage as if you put your hand in a 900W microwave oven
– Suppen
yesterday




18




18




This is an odd question. Let me pose a question to you: A lead bullet is much more dense than a grand piano, and yet if you drop a grand piano on me from ten meters up, it will be much more harmful than if you drop a lead bullet on me from ten meters up. So are pianos more dangerous than bullets despite their low density? ("Should we have better piano control?" is a political question of course.)
– Eric Lippert
yesterday






This is an odd question. Let me pose a question to you: A lead bullet is much more dense than a grand piano, and yet if you drop a grand piano on me from ten meters up, it will be much more harmful than if you drop a lead bullet on me from ten meters up. So are pianos more dangerous than bullets despite their low density? ("Should we have better piano control?" is a political question of course.)
– Eric Lippert
yesterday












12 Answers
12






active

oldest

votes


















80














Your question contains a premise that is false: Microwaves do not have less energy than visible light per se. They only have less energy per photon, as per the Planck–Einstein relation, $E = hf$. In other words, you can raise the power of electromagnetic radiation to a dangerous level at any wavelength, if only you generate enough photons – as your microwave oven does.



That very much includes visible light. You can easily verify this by waiting for a sunny day, getting out your magnifying glass, and using it to focus sunlight on a piece of paper. Watch it char and maybe even burn. (Make sure there's nothing around that piece of paper that can burn.) In conclusion, then, sunlight is dangerous!






share|cite|improve this answer










New contributor




Thomas Blankenhorn is a new contributor to this site. Take care in asking for clarification, commenting, and answering.
Check out our Code of Conduct.














  • 18




    "You can easily verify this by waiting for a sunny day, getting out your magnifying glass, and using it to focus sunlight onto your forearm." Nope. Don't do this. It's not good for you, unless you like burns, sunburn and melanoma.
    – wizzwizz4
    yesterday






  • 7




    I was half-joking, and wizzwizz4 makes a fair point. While the experiment does demonstrate that visible light can be harmful, the flip side is, don't do it if you don't like pain! No to the specific point about sunburn and melanoma, though. These are both caused by ionizing radiation (ultraviolet and shorter wavelengths),. Most of that gets filtered out by the glass (or plastic) that lenses are made of. During the few seconds until one feels the burn, twitches, and thereby ends the experiment, one won't accumulate enough to cause lasting harm.
    – Thomas Blankenhorn
    yesterday








  • 27




    Safety comment - For any readers who may not realize this - The note about burning hair is not a joke. If you focus sunlight on your skin with a magnifying glass, you will burn yourself severely. DO NOT DO THIS.
    – TypeIA
    yesterday






  • 9




    You may want to suggest that they try the experiment on something like a piece of paper first. You can quickly figure out a good reason on your own for why you wouldn't want to focus it on your arm.
    – JMac
    yesterday






  • 3




    Safety comment: don't stare at the Sun with a magnifying glass especially when our eyes are exactly placed on the focal point.
    – God Must Be Crazy
    yesterday



















42














If you stare at the Sun you’ll go blind. And if you spend a lot of time in the sun, you’re likely to get skin cancers. So visible light seems plenty dangerous to me.



Some of the damage may actually be from infrared and ultraviolet light, but these are close in frequency to visible light and very far from microwaves.



By the way, the intensity also matters, not just the frequency. In terms of photons, it matters not only how energetic each photon is, but also how many photons are arriving per second.






share|cite|improve this answer

















  • 16




    not to forget laser guns,
    – anna v
    2 days ago






  • 22




    This answer could be improved by addressing the OP's comparison with microwaves and discussing ways in which microwaves are dangerous (heating) and are not (cancer).
    – Ben Crowell
    yesterday






  • 4




    Could also compare the wattage of a microwave vs sunlight
    – JollyJoker
    yesterday








  • 6




    @BenCrowell The thing is, there's nothing really dangerous about microwaves - the only damage they do is through heating, which doesn't depend on the wavelength as long as the material absorbs most of the energy anyway. The same amount of energy from visible light does far more damage. But adding it to the answer is probably a good idea, since it seems suse doesn't understand that part.
    – Luaan
    yesterday






  • 12




    "And if you spend a lot of time in the sun, you’re likely to get skin cancers. So visible light seems plenty dangerous to me." - Skin cancer is caused by the UV radiation the sun emits, not visible light.
    – marcelm
    yesterday



















35














The dose (or, in this case, the intensity) makes the poison. You're constantly exposed to microwaves since that's what 99% of wireless communication devices use, and you're also constantly exposed to visible light unless you sleep in an isolation tank. Both can be dangerous if you increase the intensity sufficiently.






share|cite|improve this answer

















  • 1




    For that matter, you are constantly exposed to microwaves from the sun and, for that matter, the CMB. Just at very low levels.
    – WhatRoughBeast
    yesterday






  • 4




    Let me put some numbers behind that. A standard microwave oven puts 650W of electromagnetic energy into whatever you put inside it (many ovens actually do more these days). A wifi router is limited to 4 watts EIRP of RF radiation, which falls off with distance on the familiar inverse-square law. A modern bicycle headlamp, operating from a 3-watt dynamo, uses LEDs with a thermal efficiency of about 50%, so emits about 1.5 watts of visible light energy - and that's pretty bright. Now imagine looking at 650W of visible light…
    – Chromatix
    18 hours ago






  • 1




    For that matter, constantly sleeping in an isolation tank and never seeing any light whatsoever is also dangerous, but for entirely different reasons.
    – gerrit
    16 hours ago



















17














Assuming the intensity is the same, microwaves are more dangerous than visible light because they penetrate the skin to a greater depth (1-2 cm; more info is in Wikipedia).



Humans have more adaptation to visible light than to microwave radiation, because they were exposed to light for millions of years. This is expressed in two ways:




  • Proteins in the epidermis (outer skin layer) are more resistant to heat than those in the deeper skin layers

  • There are more nerve endings in the outer layers of skin, so dangerous heating by visible light causes more pain, urging you to escape the dangerous situation


Oh, and the most obvious difference: visible light is visible. Dangerous levels of visible light (e.g. in a solar cooker), to our eyes, look blinding and obviously dangerous. Dangerous levels of microwave radiation are invisible.






share|cite|improve this answer























  • In general, anything that penetrates more is less dangerous; since it means it's less ionising. For example, gamma radiation; when it does ionise does a whole lot of damage... however it's such high energy, most of them are able to pass through the body without impacting it at all. While something like an alpha particle (sure, it's not EM, but concept applies) becomes VERY dangerous because it doesn't penetrate at all. Obviously, reflecting and passing through are the same in that the energy isn't absorbed by the organs; meaning no ionisation occurs.
    – UKMonkey
    yesterday








  • 1




    Wikipedia doesn't seem to support your claim. For instance, the microwave intensity found to cause cataracts in rabbits is 150 mW/cm2 for 100 minutes. That is roughly equal to the intensity of natural sunlight. I believe staring at the Sun will make you blind much faster.
    – Dmitry Grigoryev
    yesterday






  • 3




    @UKMonkey Re "alpha is more dangerous than gamma because it penetrates less": Simply not true: "exposure to most alpha particles originating outside the body is not a serious hazard." Or maybe a Briton would trust the BBC. The same applies to light: The skin absorbs/reflects lots of it and protects underlying tissue; while microwaves can penetrate it and reach living cells.
    – Peter A. Schneider
    yesterday












  • @PeterA.Schneider Sure - emphasis on outside the body. Inside the body is a very different story. That's because our skin has been very well adapted to protect us. The point is that you can't just say "it's more penetrating -> it's more dangerous" EM Radiation isn't a gun - if it goes through you, it's harmless. Now there's a sweet spot where all the radiation is absorbed in the body, and it penetrates some distance - which is how cancer treatment works; however attempting to suggest that masters in physics is related to the media is insulting, and I'd be thankful if you could refrain.
    – UKMonkey
    yesterday






  • 1




    I can vouch for the heating effect of visible light on skin. A 2kW theatrical followspot with an IR-reducing coating (so probably 100-200W in 15-20cm diameter) has a beam powerful enough that you wouldn't leave your hand in it for long or choose to put it back in. For pure visible light, even a couple of hundred mW of 532nm laser (parallel beam) certainly stings enough to deter longer-term testing
    – Chris H
    yesterday



















15














The danger of electromagnetic waves is a function of the photon energy, the intensity of the source and your distance from it, and the qualitative nature of the interaction of a specific frequency with organic matter.



That latter bit is very complex. The visible spectrum, down to about infra-red, doesn't penetrate the top layer of skin, or most clothing, so for the most part its interaction is limited to heating. Strong infrared can certainly cause burns. Strong visible light can certainly cause eye damage. But very high-intensity sources of visible light are rare in daily life, and very notably, we can see them, and avoid them.



Further down in the energy spectrum (longer wavelength) you get the "millimeter waves" of airport scanners, which can penetrate clothing but not skin, and then you're into the microwaves, uhf and vfh radio waves, and then the radio waves called shortwaves (high frequency), medium waves and longwaves (low frequency). Microwaves can penetrate into flesh, and radio frequencies can entirely traverse a human body, and these can cause very, very severe deep-tissue burns. Certain frequencies can also interfere with cardiac rhythm, which can be as fatal as it sounds.



Your home wifi equipment produces microwaves on pretty much the same frequencies as your microwave oven, but at milliwatt power and dispersed in all directions. The oven dumps hundreds of watts into a small enclosed space. That's the difference.



And we live in a sea of radio waves from microwave (cell phone, wifi) though uhf and vhf (two-way radios, broadcast TV and FM) and lower (broadcast radio). The key is the power. If you grab the antenna of your uncle's 500-W ham radio when he keys the mic, or climb the tower of a multi-kilowatt tv station, you'll get hurt, maybe very badly. But going about your normal business, you're probably absorbing less than a milliwatt of radio energy. And the only effect is heating, so it's little different than being in a room that's very slightly warmer.



Now, moving upward in energy from the visible spectrum, you get ultraviolet, x-rays and then a vast spectrum of increasing energy gamma. Not only can they penetrate into flesh, they have a very specific dirty trick: they have enough photon energy to ionize molecules, and when that happens to our dna and proteins, we start to have very bad days. This is a very specific capability that begins at a certain energy threshold.



Microwaves, as you've remarked, are in the opposite direction from UV, X-rays and gamma: lower photon energy, longer wavelength. They cannot duplicate the ionization danger of higher energies, no matter how intense their sources are.






share|cite|improve this answer








New contributor




CCTO is a new contributor to this site. Take care in asking for clarification, commenting, and answering.
Check out our Code of Conduct.














  • 1




    Certain frequencies can also interfere with cardiac rhythm, can you elaborate on that? I'm aware of the case of pacemakers, but how would this work in the fully natural situation? If it's not resonance you're talking about, then what is it?
    – gerrit
    16 hours ago





















8














Other answers already point out the matter of intensity. If you have a 1kW microwave that cooks your chicken and so call microwaves dangerous, you can equally cook chicken with 1kW visible light bulb. The difference is mostly how well and how deep the absorption goes, but the amount of energy is the same if the absorbed power is the same.



What makes the difference:




  • Absorption coefficient; how WELL the light is absorbed. So, a black chicken will cook well in strong visible light, but a white chicken will require higher power, because it reflects more. Microwaves work well because they penetrate deeper before they get absorbed (due to longer wavelength), but also absorb WELL because the frequency is tuned near the resonance for water molecules.

  • Resonances; If the wavelength matches exactly to one of the transition frequencies for molecules/atoms in the target, most of the energy is selectively absorbed just by those molecules. So, if you tuned your light specifically to a transition that breaks some specific bonds, or heat just specific tissues. This can do more damage because it may change chemistry, but luckily, breaking bonds requires quite high frequencies - see next case below. With MW and IR, you will still end up just heating the sample if you find a resonance (resonances in MW, IR and visible are mostly vibrational and rotational transitions, not bond changing, except for red-colored substances reacting to visible light, which you notice when red dyes bleach quickly in intense light).

  • Ionization; If the energy of a SINGLE photon is enough to kick off an electron from a molecule/atom, then it's dangerous because it's actively affecting the chemistry (note that this is similar result than the resonant case above, but instead of having a precise frequency, it has way too much energy, with similar results). This is what is called ionising radiation (gamma/X-rays, down to UV range).


Note that resonance just means good absorbtion, nothing mystical. Water is mostly transparent for visible light because no significant vibrations of water molecule fall in this range - most are in IR and microwave, and there is another absorption range in UV.



Rule of thumb: microwaves, IR and visible light just heat you up. It's the heat is sufficient to raise the temperature into danger zone, it's dangerous, otherwise it's harmless. Only intensity matters (Watts per square meter), not the frequency.
Ionizing radiation (UV/X-ray/gamma) are dangerous because of chemical damage even at low intensity.



Microwaves are NOT ionizing radiation, so the wireless and mobile signals do absolutely nothing - the power is way too low, otherwise you'd need to charge your phone every 5 minutes.






share|cite|improve this answer





























    5














    It's more accurate to say microwave ovens are dangerous. Then again, so is visible light.



    It's a question not of photon energy, but total energy. A typical microwave outputs on the order of 1 kilowatt of electromagnetic radiation which is almost entirely absorbed by the food within.



    By comparison, the solar power at earth's surface, at maximum, is around 1 kilowatt per square meter. If it's cloudy, not at the equator, or not noon, it will be less. Most foods have a surface area of much less than a square meter, so the total electromagnetic radiation power received by something sitting in the sun is much less than a microwave oven.



    For a fair comparison, what do you think would happen if a magnifying glass with an area of one square meter, on a very sunny day, were used to focus light on to something the size of what you'd put into a microwave oven?



    There are a few more subtle differences. For example dangerous powers of visible light are so bright you'll surely close your eyes. Furthermore, visible light penetrates less deeply, so you're likely to feel the heat and move away before it does any more than superficial damage to your skin, like a sunburn. On the other hand, microwave radiation is invisible and penetrates more deeply, so you may suffer irreversible injury before even noticing the hazard. The cornea is especially prone to microwave injury since there's no protective reflex to protect it, it has little thermal mass and thus heats quickly, and there's little blood flow to cool it.






    share|cite|improve this answer























    • Of course, solar ovens are a thing - and indeed, to get a decent oven, a square meter of mirrors concentrating the sunlight to the center (with the food item) is more than enough.
      – Luaan
      20 hours ago



















    1














    There is a saying that "The dose makes the toxin."



    Oxygen is the substance you most need a constant supply of. You will die after just a few minutes without oxygen.



    But oxygen toxicity is real. Too much oxygen can harm or kill you. In fact, for billions of years all the organisms on Earth had no use for oxygen. When oxygen concentration increased in the atmosphere most life forms on Earth died off. Only the ones that adapted fast enough to tolerate and even to depend on oxygen could survive in the more oxygen rich atmosphere.



    The same goes for every other necessary substance or environmental factor. And the same goes for every other dangerous substance or environmental factor. In high enough doses, even the most necessary things are deadly. In low enough doses, even the most deadly things can be harmless and maybe even useful.



    Since visible light and all other frequencies of electromagnetic radiation are environmental factors, the preceding is also true for them. Too much of any frequencies of electromagnetic radiation, even the most beneficial, can be harmful or deadly, and small enough exposure to even the deadliest frequencies of electromagnetic radiation, such as X-Rays or gamma rays, can be harmless or even beneficial.



    I remember a story by Arthur C. Clarke in which a character criticized the way that death rays in science fiction were visible to the human eye, saying that if visible light was deadly, humans couldn't live. But humans have evolved to survive the concentrations of visible light that are common on Earth. A human exposed to a concentration of visible light that was a thousand times, or a million times, or a billion times, stronger could be killed, cooked, or even instantly vaporized.



    I also remember two other stories by Arthur C. Clarke, perhaps even in the same collection, where humans found plausible ways to create death rays out of visible light using the primitive technology of the 1950s and 1960s.



    Some forms of radiation therapy for cancer involve using beams of X-Rays, Gamma rays, or charged particles to help kill cancer cells. So people undergoing radiation therapy are often benefited by being struck by death rays designed to kill living tissues, because the death rays are aimed at living tissues that would kill their host bodies eventually.



    As we all know, antimatter is the most dangerous substance imagined by physicists. If a normal particle collides with its opposite antiparticle, both are annihilated and radiation is emitted.



    You may have heard of people having PET scans for medical diagnosis. PET stands for Positron Emission Tomography. A positron is an anti electron, and thus an antiparticle. So people who had PET scans have survived, and perhaps benefited from, having minute amounts of antiparticles in their bodies.



    So even with something as supremely deadly as antimatter, the dose makes the toxin.



    A microwave oven does to food something very similar to what camp fires and stoves do to food, and requires about the same amount of energy per meal. The amount of energy received per second of being microwaved is many times the amount of energy per second in natural or artificial light for illumination. much



    So being exposed to the same energy in visible light frequencies as an open fire or a stove imparts to a meal is not likely to be much better for someone than being microwaved in a microwave oven would be.






    share|cite|improve this answer








    New contributor




    M.A. Golding is a new contributor to this site. Take care in asking for clarification, commenting, and answering.
    Check out our Code of Conduct.


























      0














      When we say that microwaves are less energetic, we are talking about the energy in a single photon. The number of photons is also important.



      A single microwave photon is utterly harmless. Its only effect is heat, and it takes a serious amount of heat to damage us. But enough heat, in any form, will kill.



      Visible light has enough energy that single photons can cause chemical reactions, but only in sensitive compounds. That is what happens in our eyes. The chemicals in our eyes are carefully constructed to be extra sensitive to light and that is what makes light visible.



      Ultraviolet light is worse. Here the photons carry enough energy to cause unwanted chemical reactions in most organic compounds. Sun burn and skin cancer results.



      Gamma ray photons from radioactivity are even worse, but they are fortunately rare.






      share|cite|improve this answer





























        0














        There is nothing inherently more or less dangerous about microwaves. Yes, the type of damage via microwave vs. visible light vs. xray are different but weather or not the light cause damage has the same factor for all the spectrum of light - intensity.



        Remember, modern computer-using humans constantly volunteer to be bather in microwaves. Wifi uses exactly the same frequency as microwave ovens. The difference between wifi and microwave ovens is the wattage - the amount of power used to generate the light - the intensity.



        You can cook with visible light if you pump enough power into it - or somehow concentrate it. This is how sun ovens work and how you can burn paper with a magnifying glass. You can also cook with xrays if it is intense enough.



        Side note: Most people don't realise this but high intensity infrared light can blind you as surely as staring at the sun. Just because it is invisible to your eyes does not mean the photons does not hit your retina. High intensity infrared spotlights are sold as part of security systems for infrared sensitive cameras (night vision).






        share|cite|improve this answer





















        • "High intensity" is relative, of course; most spotlights used for "night vision cameras" are still just a few watts, which is rather small compared to sunlight. There is a danger mainly because you don't realize you're too close to the light (and there's no eye response to the increasing amount of light hitting your eyes), and if it's dark, your pupils are maximally dilated. You don't see microwaves, but above about 20 W, the heat is quite noticeable, and a 200 W light is unmistakably warm (those are usually used for heating, though, not IR camera illumination).
          – Luaan
          20 hours ago



















        -1














        The trick with microwaves is that they use resonance frequencies of water. Some microwaves can be tuned to meat, vegetables or fish, since the resonance frequency can change slightly in composition. Visible light does not resonate with anything in our bodies.



        Our generally photo-sensitive skin mostly reacts to UV-Ranges, even in a not yet ionizing spectrum. (The wavelength also determines how deep the light can enter the skin!)



        With enough energy, visible light could be dangerous, but the energy needed is far greater without any resonance effects. (think of a child swing)






        share|cite|improve this answer








        New contributor




        Anonymous is a new contributor to this site. Take care in asking for clarification, commenting, and answering.
        Check out our Code of Conduct.


















        • Welcome to Physics SE! Hope you'll enjoy it here. Great first answer.
          – Andrea
          yesterday






        • 12




          Microwaves don't use resonance; it's a common misconception. See explanation e.g. here or anywhere on the Internet.
          – user27542
          yesterday






        • 1




          In what is fish water different from meat water or vegetable water? Not to mention that you can heat up butter just fine in a microwave.
          – Dmitry Grigoryev
          yesterday








        • 1




          @DmitryGrigoryev -- butter has a lot of water in it. When you put butter in a hot pan, the bubbles are the water boiling off.
          – Pete Becker
          yesterday








        • 1




          I think resonance is the wrong word here. I think the more correct phrasing is that most microwave ovens emit at a frequency that corresponds to a peak in the absorption spectrum of water. This answer is right in noting that absorption is another key factor in addition to the energy per photon and the intensity of light.
          – Cogitator
          yesterday



















        -1














        Stand in front of a 2KW spotlight, like used in stage productions.
        You will start to feel it on your skin (or simply blind you). As explained elsewhere, microwaves go deeper and a microwave oven is still delivering a KW of energy to your body.






        share|cite|improve this answer








        New contributor




        Reed Shilts is a new contributor to this site. Take care in asking for clarification, commenting, and answering.
        Check out our Code of Conduct.

















          protected by rob 5 hours ago



          Thank you for your interest in this question.
          Because it has attracted low-quality or spam answers that had to be removed, posting an answer now requires 10 reputation on this site (the association bonus does not count).



          Would you like to answer one of these unanswered questions instead?














          12 Answers
          12






          active

          oldest

          votes








          12 Answers
          12






          active

          oldest

          votes









          active

          oldest

          votes






          active

          oldest

          votes









          80














          Your question contains a premise that is false: Microwaves do not have less energy than visible light per se. They only have less energy per photon, as per the Planck–Einstein relation, $E = hf$. In other words, you can raise the power of electromagnetic radiation to a dangerous level at any wavelength, if only you generate enough photons – as your microwave oven does.



          That very much includes visible light. You can easily verify this by waiting for a sunny day, getting out your magnifying glass, and using it to focus sunlight on a piece of paper. Watch it char and maybe even burn. (Make sure there's nothing around that piece of paper that can burn.) In conclusion, then, sunlight is dangerous!






          share|cite|improve this answer










          New contributor




          Thomas Blankenhorn is a new contributor to this site. Take care in asking for clarification, commenting, and answering.
          Check out our Code of Conduct.














          • 18




            "You can easily verify this by waiting for a sunny day, getting out your magnifying glass, and using it to focus sunlight onto your forearm." Nope. Don't do this. It's not good for you, unless you like burns, sunburn and melanoma.
            – wizzwizz4
            yesterday






          • 7




            I was half-joking, and wizzwizz4 makes a fair point. While the experiment does demonstrate that visible light can be harmful, the flip side is, don't do it if you don't like pain! No to the specific point about sunburn and melanoma, though. These are both caused by ionizing radiation (ultraviolet and shorter wavelengths),. Most of that gets filtered out by the glass (or plastic) that lenses are made of. During the few seconds until one feels the burn, twitches, and thereby ends the experiment, one won't accumulate enough to cause lasting harm.
            – Thomas Blankenhorn
            yesterday








          • 27




            Safety comment - For any readers who may not realize this - The note about burning hair is not a joke. If you focus sunlight on your skin with a magnifying glass, you will burn yourself severely. DO NOT DO THIS.
            – TypeIA
            yesterday






          • 9




            You may want to suggest that they try the experiment on something like a piece of paper first. You can quickly figure out a good reason on your own for why you wouldn't want to focus it on your arm.
            – JMac
            yesterday






          • 3




            Safety comment: don't stare at the Sun with a magnifying glass especially when our eyes are exactly placed on the focal point.
            – God Must Be Crazy
            yesterday
















          80














          Your question contains a premise that is false: Microwaves do not have less energy than visible light per se. They only have less energy per photon, as per the Planck–Einstein relation, $E = hf$. In other words, you can raise the power of electromagnetic radiation to a dangerous level at any wavelength, if only you generate enough photons – as your microwave oven does.



          That very much includes visible light. You can easily verify this by waiting for a sunny day, getting out your magnifying glass, and using it to focus sunlight on a piece of paper. Watch it char and maybe even burn. (Make sure there's nothing around that piece of paper that can burn.) In conclusion, then, sunlight is dangerous!






          share|cite|improve this answer










          New contributor




          Thomas Blankenhorn is a new contributor to this site. Take care in asking for clarification, commenting, and answering.
          Check out our Code of Conduct.














          • 18




            "You can easily verify this by waiting for a sunny day, getting out your magnifying glass, and using it to focus sunlight onto your forearm." Nope. Don't do this. It's not good for you, unless you like burns, sunburn and melanoma.
            – wizzwizz4
            yesterday






          • 7




            I was half-joking, and wizzwizz4 makes a fair point. While the experiment does demonstrate that visible light can be harmful, the flip side is, don't do it if you don't like pain! No to the specific point about sunburn and melanoma, though. These are both caused by ionizing radiation (ultraviolet and shorter wavelengths),. Most of that gets filtered out by the glass (or plastic) that lenses are made of. During the few seconds until one feels the burn, twitches, and thereby ends the experiment, one won't accumulate enough to cause lasting harm.
            – Thomas Blankenhorn
            yesterday








          • 27




            Safety comment - For any readers who may not realize this - The note about burning hair is not a joke. If you focus sunlight on your skin with a magnifying glass, you will burn yourself severely. DO NOT DO THIS.
            – TypeIA
            yesterday






          • 9




            You may want to suggest that they try the experiment on something like a piece of paper first. You can quickly figure out a good reason on your own for why you wouldn't want to focus it on your arm.
            – JMac
            yesterday






          • 3




            Safety comment: don't stare at the Sun with a magnifying glass especially when our eyes are exactly placed on the focal point.
            – God Must Be Crazy
            yesterday














          80












          80








          80






          Your question contains a premise that is false: Microwaves do not have less energy than visible light per se. They only have less energy per photon, as per the Planck–Einstein relation, $E = hf$. In other words, you can raise the power of electromagnetic radiation to a dangerous level at any wavelength, if only you generate enough photons – as your microwave oven does.



          That very much includes visible light. You can easily verify this by waiting for a sunny day, getting out your magnifying glass, and using it to focus sunlight on a piece of paper. Watch it char and maybe even burn. (Make sure there's nothing around that piece of paper that can burn.) In conclusion, then, sunlight is dangerous!






          share|cite|improve this answer










          New contributor




          Thomas Blankenhorn is a new contributor to this site. Take care in asking for clarification, commenting, and answering.
          Check out our Code of Conduct.









          Your question contains a premise that is false: Microwaves do not have less energy than visible light per se. They only have less energy per photon, as per the Planck–Einstein relation, $E = hf$. In other words, you can raise the power of electromagnetic radiation to a dangerous level at any wavelength, if only you generate enough photons – as your microwave oven does.



          That very much includes visible light. You can easily verify this by waiting for a sunny day, getting out your magnifying glass, and using it to focus sunlight on a piece of paper. Watch it char and maybe even burn. (Make sure there's nothing around that piece of paper that can burn.) In conclusion, then, sunlight is dangerous!







          share|cite|improve this answer










          New contributor




          Thomas Blankenhorn is a new contributor to this site. Take care in asking for clarification, commenting, and answering.
          Check out our Code of Conduct.









          share|cite|improve this answer



          share|cite|improve this answer








          edited 9 hours ago









          mkrieger1

          1033




          1033






          New contributor




          Thomas Blankenhorn is a new contributor to this site. Take care in asking for clarification, commenting, and answering.
          Check out our Code of Conduct.









          answered yesterday









          Thomas BlankenhornThomas Blankenhorn

          58127




          58127




          New contributor




          Thomas Blankenhorn is a new contributor to this site. Take care in asking for clarification, commenting, and answering.
          Check out our Code of Conduct.





          New contributor





          Thomas Blankenhorn is a new contributor to this site. Take care in asking for clarification, commenting, and answering.
          Check out our Code of Conduct.






          Thomas Blankenhorn is a new contributor to this site. Take care in asking for clarification, commenting, and answering.
          Check out our Code of Conduct.








          • 18




            "You can easily verify this by waiting for a sunny day, getting out your magnifying glass, and using it to focus sunlight onto your forearm." Nope. Don't do this. It's not good for you, unless you like burns, sunburn and melanoma.
            – wizzwizz4
            yesterday






          • 7




            I was half-joking, and wizzwizz4 makes a fair point. While the experiment does demonstrate that visible light can be harmful, the flip side is, don't do it if you don't like pain! No to the specific point about sunburn and melanoma, though. These are both caused by ionizing radiation (ultraviolet and shorter wavelengths),. Most of that gets filtered out by the glass (or plastic) that lenses are made of. During the few seconds until one feels the burn, twitches, and thereby ends the experiment, one won't accumulate enough to cause lasting harm.
            – Thomas Blankenhorn
            yesterday








          • 27




            Safety comment - For any readers who may not realize this - The note about burning hair is not a joke. If you focus sunlight on your skin with a magnifying glass, you will burn yourself severely. DO NOT DO THIS.
            – TypeIA
            yesterday






          • 9




            You may want to suggest that they try the experiment on something like a piece of paper first. You can quickly figure out a good reason on your own for why you wouldn't want to focus it on your arm.
            – JMac
            yesterday






          • 3




            Safety comment: don't stare at the Sun with a magnifying glass especially when our eyes are exactly placed on the focal point.
            – God Must Be Crazy
            yesterday














          • 18




            "You can easily verify this by waiting for a sunny day, getting out your magnifying glass, and using it to focus sunlight onto your forearm." Nope. Don't do this. It's not good for you, unless you like burns, sunburn and melanoma.
            – wizzwizz4
            yesterday






          • 7




            I was half-joking, and wizzwizz4 makes a fair point. While the experiment does demonstrate that visible light can be harmful, the flip side is, don't do it if you don't like pain! No to the specific point about sunburn and melanoma, though. These are both caused by ionizing radiation (ultraviolet and shorter wavelengths),. Most of that gets filtered out by the glass (or plastic) that lenses are made of. During the few seconds until one feels the burn, twitches, and thereby ends the experiment, one won't accumulate enough to cause lasting harm.
            – Thomas Blankenhorn
            yesterday








          • 27




            Safety comment - For any readers who may not realize this - The note about burning hair is not a joke. If you focus sunlight on your skin with a magnifying glass, you will burn yourself severely. DO NOT DO THIS.
            – TypeIA
            yesterday






          • 9




            You may want to suggest that they try the experiment on something like a piece of paper first. You can quickly figure out a good reason on your own for why you wouldn't want to focus it on your arm.
            – JMac
            yesterday






          • 3




            Safety comment: don't stare at the Sun with a magnifying glass especially when our eyes are exactly placed on the focal point.
            – God Must Be Crazy
            yesterday








          18




          18




          "You can easily verify this by waiting for a sunny day, getting out your magnifying glass, and using it to focus sunlight onto your forearm." Nope. Don't do this. It's not good for you, unless you like burns, sunburn and melanoma.
          – wizzwizz4
          yesterday




          "You can easily verify this by waiting for a sunny day, getting out your magnifying glass, and using it to focus sunlight onto your forearm." Nope. Don't do this. It's not good for you, unless you like burns, sunburn and melanoma.
          – wizzwizz4
          yesterday




          7




          7




          I was half-joking, and wizzwizz4 makes a fair point. While the experiment does demonstrate that visible light can be harmful, the flip side is, don't do it if you don't like pain! No to the specific point about sunburn and melanoma, though. These are both caused by ionizing radiation (ultraviolet and shorter wavelengths),. Most of that gets filtered out by the glass (or plastic) that lenses are made of. During the few seconds until one feels the burn, twitches, and thereby ends the experiment, one won't accumulate enough to cause lasting harm.
          – Thomas Blankenhorn
          yesterday






          I was half-joking, and wizzwizz4 makes a fair point. While the experiment does demonstrate that visible light can be harmful, the flip side is, don't do it if you don't like pain! No to the specific point about sunburn and melanoma, though. These are both caused by ionizing radiation (ultraviolet and shorter wavelengths),. Most of that gets filtered out by the glass (or plastic) that lenses are made of. During the few seconds until one feels the burn, twitches, and thereby ends the experiment, one won't accumulate enough to cause lasting harm.
          – Thomas Blankenhorn
          yesterday






          27




          27




          Safety comment - For any readers who may not realize this - The note about burning hair is not a joke. If you focus sunlight on your skin with a magnifying glass, you will burn yourself severely. DO NOT DO THIS.
          – TypeIA
          yesterday




          Safety comment - For any readers who may not realize this - The note about burning hair is not a joke. If you focus sunlight on your skin with a magnifying glass, you will burn yourself severely. DO NOT DO THIS.
          – TypeIA
          yesterday




          9




          9




          You may want to suggest that they try the experiment on something like a piece of paper first. You can quickly figure out a good reason on your own for why you wouldn't want to focus it on your arm.
          – JMac
          yesterday




          You may want to suggest that they try the experiment on something like a piece of paper first. You can quickly figure out a good reason on your own for why you wouldn't want to focus it on your arm.
          – JMac
          yesterday




          3




          3




          Safety comment: don't stare at the Sun with a magnifying glass especially when our eyes are exactly placed on the focal point.
          – God Must Be Crazy
          yesterday




          Safety comment: don't stare at the Sun with a magnifying glass especially when our eyes are exactly placed on the focal point.
          – God Must Be Crazy
          yesterday











          42














          If you stare at the Sun you’ll go blind. And if you spend a lot of time in the sun, you’re likely to get skin cancers. So visible light seems plenty dangerous to me.



          Some of the damage may actually be from infrared and ultraviolet light, but these are close in frequency to visible light and very far from microwaves.



          By the way, the intensity also matters, not just the frequency. In terms of photons, it matters not only how energetic each photon is, but also how many photons are arriving per second.






          share|cite|improve this answer

















          • 16




            not to forget laser guns,
            – anna v
            2 days ago






          • 22




            This answer could be improved by addressing the OP's comparison with microwaves and discussing ways in which microwaves are dangerous (heating) and are not (cancer).
            – Ben Crowell
            yesterday






          • 4




            Could also compare the wattage of a microwave vs sunlight
            – JollyJoker
            yesterday








          • 6




            @BenCrowell The thing is, there's nothing really dangerous about microwaves - the only damage they do is through heating, which doesn't depend on the wavelength as long as the material absorbs most of the energy anyway. The same amount of energy from visible light does far more damage. But adding it to the answer is probably a good idea, since it seems suse doesn't understand that part.
            – Luaan
            yesterday






          • 12




            "And if you spend a lot of time in the sun, you’re likely to get skin cancers. So visible light seems plenty dangerous to me." - Skin cancer is caused by the UV radiation the sun emits, not visible light.
            – marcelm
            yesterday
















          42














          If you stare at the Sun you’ll go blind. And if you spend a lot of time in the sun, you’re likely to get skin cancers. So visible light seems plenty dangerous to me.



          Some of the damage may actually be from infrared and ultraviolet light, but these are close in frequency to visible light and very far from microwaves.



          By the way, the intensity also matters, not just the frequency. In terms of photons, it matters not only how energetic each photon is, but also how many photons are arriving per second.






          share|cite|improve this answer

















          • 16




            not to forget laser guns,
            – anna v
            2 days ago






          • 22




            This answer could be improved by addressing the OP's comparison with microwaves and discussing ways in which microwaves are dangerous (heating) and are not (cancer).
            – Ben Crowell
            yesterday






          • 4




            Could also compare the wattage of a microwave vs sunlight
            – JollyJoker
            yesterday








          • 6




            @BenCrowell The thing is, there's nothing really dangerous about microwaves - the only damage they do is through heating, which doesn't depend on the wavelength as long as the material absorbs most of the energy anyway. The same amount of energy from visible light does far more damage. But adding it to the answer is probably a good idea, since it seems suse doesn't understand that part.
            – Luaan
            yesterday






          • 12




            "And if you spend a lot of time in the sun, you’re likely to get skin cancers. So visible light seems plenty dangerous to me." - Skin cancer is caused by the UV radiation the sun emits, not visible light.
            – marcelm
            yesterday














          42












          42








          42






          If you stare at the Sun you’ll go blind. And if you spend a lot of time in the sun, you’re likely to get skin cancers. So visible light seems plenty dangerous to me.



          Some of the damage may actually be from infrared and ultraviolet light, but these are close in frequency to visible light and very far from microwaves.



          By the way, the intensity also matters, not just the frequency. In terms of photons, it matters not only how energetic each photon is, but also how many photons are arriving per second.






          share|cite|improve this answer












          If you stare at the Sun you’ll go blind. And if you spend a lot of time in the sun, you’re likely to get skin cancers. So visible light seems plenty dangerous to me.



          Some of the damage may actually be from infrared and ultraviolet light, but these are close in frequency to visible light and very far from microwaves.



          By the way, the intensity also matters, not just the frequency. In terms of photons, it matters not only how energetic each photon is, but also how many photons are arriving per second.







          share|cite|improve this answer












          share|cite|improve this answer



          share|cite|improve this answer










          answered 2 days ago









          G. SmithG. Smith

          4,9181021




          4,9181021








          • 16




            not to forget laser guns,
            – anna v
            2 days ago






          • 22




            This answer could be improved by addressing the OP's comparison with microwaves and discussing ways in which microwaves are dangerous (heating) and are not (cancer).
            – Ben Crowell
            yesterday






          • 4




            Could also compare the wattage of a microwave vs sunlight
            – JollyJoker
            yesterday








          • 6




            @BenCrowell The thing is, there's nothing really dangerous about microwaves - the only damage they do is through heating, which doesn't depend on the wavelength as long as the material absorbs most of the energy anyway. The same amount of energy from visible light does far more damage. But adding it to the answer is probably a good idea, since it seems suse doesn't understand that part.
            – Luaan
            yesterday






          • 12




            "And if you spend a lot of time in the sun, you’re likely to get skin cancers. So visible light seems plenty dangerous to me." - Skin cancer is caused by the UV radiation the sun emits, not visible light.
            – marcelm
            yesterday














          • 16




            not to forget laser guns,
            – anna v
            2 days ago






          • 22




            This answer could be improved by addressing the OP's comparison with microwaves and discussing ways in which microwaves are dangerous (heating) and are not (cancer).
            – Ben Crowell
            yesterday






          • 4




            Could also compare the wattage of a microwave vs sunlight
            – JollyJoker
            yesterday








          • 6




            @BenCrowell The thing is, there's nothing really dangerous about microwaves - the only damage they do is through heating, which doesn't depend on the wavelength as long as the material absorbs most of the energy anyway. The same amount of energy from visible light does far more damage. But adding it to the answer is probably a good idea, since it seems suse doesn't understand that part.
            – Luaan
            yesterday






          • 12




            "And if you spend a lot of time in the sun, you’re likely to get skin cancers. So visible light seems plenty dangerous to me." - Skin cancer is caused by the UV radiation the sun emits, not visible light.
            – marcelm
            yesterday








          16




          16




          not to forget laser guns,
          – anna v
          2 days ago




          not to forget laser guns,
          – anna v
          2 days ago




          22




          22




          This answer could be improved by addressing the OP's comparison with microwaves and discussing ways in which microwaves are dangerous (heating) and are not (cancer).
          – Ben Crowell
          yesterday




          This answer could be improved by addressing the OP's comparison with microwaves and discussing ways in which microwaves are dangerous (heating) and are not (cancer).
          – Ben Crowell
          yesterday




          4




          4




          Could also compare the wattage of a microwave vs sunlight
          – JollyJoker
          yesterday






          Could also compare the wattage of a microwave vs sunlight
          – JollyJoker
          yesterday






          6




          6




          @BenCrowell The thing is, there's nothing really dangerous about microwaves - the only damage they do is through heating, which doesn't depend on the wavelength as long as the material absorbs most of the energy anyway. The same amount of energy from visible light does far more damage. But adding it to the answer is probably a good idea, since it seems suse doesn't understand that part.
          – Luaan
          yesterday




          @BenCrowell The thing is, there's nothing really dangerous about microwaves - the only damage they do is through heating, which doesn't depend on the wavelength as long as the material absorbs most of the energy anyway. The same amount of energy from visible light does far more damage. But adding it to the answer is probably a good idea, since it seems suse doesn't understand that part.
          – Luaan
          yesterday




          12




          12




          "And if you spend a lot of time in the sun, you’re likely to get skin cancers. So visible light seems plenty dangerous to me." - Skin cancer is caused by the UV radiation the sun emits, not visible light.
          – marcelm
          yesterday




          "And if you spend a lot of time in the sun, you’re likely to get skin cancers. So visible light seems plenty dangerous to me." - Skin cancer is caused by the UV radiation the sun emits, not visible light.
          – marcelm
          yesterday











          35














          The dose (or, in this case, the intensity) makes the poison. You're constantly exposed to microwaves since that's what 99% of wireless communication devices use, and you're also constantly exposed to visible light unless you sleep in an isolation tank. Both can be dangerous if you increase the intensity sufficiently.






          share|cite|improve this answer

















          • 1




            For that matter, you are constantly exposed to microwaves from the sun and, for that matter, the CMB. Just at very low levels.
            – WhatRoughBeast
            yesterday






          • 4




            Let me put some numbers behind that. A standard microwave oven puts 650W of electromagnetic energy into whatever you put inside it (many ovens actually do more these days). A wifi router is limited to 4 watts EIRP of RF radiation, which falls off with distance on the familiar inverse-square law. A modern bicycle headlamp, operating from a 3-watt dynamo, uses LEDs with a thermal efficiency of about 50%, so emits about 1.5 watts of visible light energy - and that's pretty bright. Now imagine looking at 650W of visible light…
            – Chromatix
            18 hours ago






          • 1




            For that matter, constantly sleeping in an isolation tank and never seeing any light whatsoever is also dangerous, but for entirely different reasons.
            – gerrit
            16 hours ago
















          35














          The dose (or, in this case, the intensity) makes the poison. You're constantly exposed to microwaves since that's what 99% of wireless communication devices use, and you're also constantly exposed to visible light unless you sleep in an isolation tank. Both can be dangerous if you increase the intensity sufficiently.






          share|cite|improve this answer

















          • 1




            For that matter, you are constantly exposed to microwaves from the sun and, for that matter, the CMB. Just at very low levels.
            – WhatRoughBeast
            yesterday






          • 4




            Let me put some numbers behind that. A standard microwave oven puts 650W of electromagnetic energy into whatever you put inside it (many ovens actually do more these days). A wifi router is limited to 4 watts EIRP of RF radiation, which falls off with distance on the familiar inverse-square law. A modern bicycle headlamp, operating from a 3-watt dynamo, uses LEDs with a thermal efficiency of about 50%, so emits about 1.5 watts of visible light energy - and that's pretty bright. Now imagine looking at 650W of visible light…
            – Chromatix
            18 hours ago






          • 1




            For that matter, constantly sleeping in an isolation tank and never seeing any light whatsoever is also dangerous, but for entirely different reasons.
            – gerrit
            16 hours ago














          35












          35








          35






          The dose (or, in this case, the intensity) makes the poison. You're constantly exposed to microwaves since that's what 99% of wireless communication devices use, and you're also constantly exposed to visible light unless you sleep in an isolation tank. Both can be dangerous if you increase the intensity sufficiently.






          share|cite|improve this answer












          The dose (or, in this case, the intensity) makes the poison. You're constantly exposed to microwaves since that's what 99% of wireless communication devices use, and you're also constantly exposed to visible light unless you sleep in an isolation tank. Both can be dangerous if you increase the intensity sufficiently.







          share|cite|improve this answer












          share|cite|improve this answer



          share|cite|improve this answer










          answered yesterday









          Dmitry GrigoryevDmitry Grigoryev

          2,8481624




          2,8481624








          • 1




            For that matter, you are constantly exposed to microwaves from the sun and, for that matter, the CMB. Just at very low levels.
            – WhatRoughBeast
            yesterday






          • 4




            Let me put some numbers behind that. A standard microwave oven puts 650W of electromagnetic energy into whatever you put inside it (many ovens actually do more these days). A wifi router is limited to 4 watts EIRP of RF radiation, which falls off with distance on the familiar inverse-square law. A modern bicycle headlamp, operating from a 3-watt dynamo, uses LEDs with a thermal efficiency of about 50%, so emits about 1.5 watts of visible light energy - and that's pretty bright. Now imagine looking at 650W of visible light…
            – Chromatix
            18 hours ago






          • 1




            For that matter, constantly sleeping in an isolation tank and never seeing any light whatsoever is also dangerous, but for entirely different reasons.
            – gerrit
            16 hours ago














          • 1




            For that matter, you are constantly exposed to microwaves from the sun and, for that matter, the CMB. Just at very low levels.
            – WhatRoughBeast
            yesterday






          • 4




            Let me put some numbers behind that. A standard microwave oven puts 650W of electromagnetic energy into whatever you put inside it (many ovens actually do more these days). A wifi router is limited to 4 watts EIRP of RF radiation, which falls off with distance on the familiar inverse-square law. A modern bicycle headlamp, operating from a 3-watt dynamo, uses LEDs with a thermal efficiency of about 50%, so emits about 1.5 watts of visible light energy - and that's pretty bright. Now imagine looking at 650W of visible light…
            – Chromatix
            18 hours ago






          • 1




            For that matter, constantly sleeping in an isolation tank and never seeing any light whatsoever is also dangerous, but for entirely different reasons.
            – gerrit
            16 hours ago








          1




          1




          For that matter, you are constantly exposed to microwaves from the sun and, for that matter, the CMB. Just at very low levels.
          – WhatRoughBeast
          yesterday




          For that matter, you are constantly exposed to microwaves from the sun and, for that matter, the CMB. Just at very low levels.
          – WhatRoughBeast
          yesterday




          4




          4




          Let me put some numbers behind that. A standard microwave oven puts 650W of electromagnetic energy into whatever you put inside it (many ovens actually do more these days). A wifi router is limited to 4 watts EIRP of RF radiation, which falls off with distance on the familiar inverse-square law. A modern bicycle headlamp, operating from a 3-watt dynamo, uses LEDs with a thermal efficiency of about 50%, so emits about 1.5 watts of visible light energy - and that's pretty bright. Now imagine looking at 650W of visible light…
          – Chromatix
          18 hours ago




          Let me put some numbers behind that. A standard microwave oven puts 650W of electromagnetic energy into whatever you put inside it (many ovens actually do more these days). A wifi router is limited to 4 watts EIRP of RF radiation, which falls off with distance on the familiar inverse-square law. A modern bicycle headlamp, operating from a 3-watt dynamo, uses LEDs with a thermal efficiency of about 50%, so emits about 1.5 watts of visible light energy - and that's pretty bright. Now imagine looking at 650W of visible light…
          – Chromatix
          18 hours ago




          1




          1




          For that matter, constantly sleeping in an isolation tank and never seeing any light whatsoever is also dangerous, but for entirely different reasons.
          – gerrit
          16 hours ago




          For that matter, constantly sleeping in an isolation tank and never seeing any light whatsoever is also dangerous, but for entirely different reasons.
          – gerrit
          16 hours ago











          17














          Assuming the intensity is the same, microwaves are more dangerous than visible light because they penetrate the skin to a greater depth (1-2 cm; more info is in Wikipedia).



          Humans have more adaptation to visible light than to microwave radiation, because they were exposed to light for millions of years. This is expressed in two ways:




          • Proteins in the epidermis (outer skin layer) are more resistant to heat than those in the deeper skin layers

          • There are more nerve endings in the outer layers of skin, so dangerous heating by visible light causes more pain, urging you to escape the dangerous situation


          Oh, and the most obvious difference: visible light is visible. Dangerous levels of visible light (e.g. in a solar cooker), to our eyes, look blinding and obviously dangerous. Dangerous levels of microwave radiation are invisible.






          share|cite|improve this answer























          • In general, anything that penetrates more is less dangerous; since it means it's less ionising. For example, gamma radiation; when it does ionise does a whole lot of damage... however it's such high energy, most of them are able to pass through the body without impacting it at all. While something like an alpha particle (sure, it's not EM, but concept applies) becomes VERY dangerous because it doesn't penetrate at all. Obviously, reflecting and passing through are the same in that the energy isn't absorbed by the organs; meaning no ionisation occurs.
            – UKMonkey
            yesterday








          • 1




            Wikipedia doesn't seem to support your claim. For instance, the microwave intensity found to cause cataracts in rabbits is 150 mW/cm2 for 100 minutes. That is roughly equal to the intensity of natural sunlight. I believe staring at the Sun will make you blind much faster.
            – Dmitry Grigoryev
            yesterday






          • 3




            @UKMonkey Re "alpha is more dangerous than gamma because it penetrates less": Simply not true: "exposure to most alpha particles originating outside the body is not a serious hazard." Or maybe a Briton would trust the BBC. The same applies to light: The skin absorbs/reflects lots of it and protects underlying tissue; while microwaves can penetrate it and reach living cells.
            – Peter A. Schneider
            yesterday












          • @PeterA.Schneider Sure - emphasis on outside the body. Inside the body is a very different story. That's because our skin has been very well adapted to protect us. The point is that you can't just say "it's more penetrating -> it's more dangerous" EM Radiation isn't a gun - if it goes through you, it's harmless. Now there's a sweet spot where all the radiation is absorbed in the body, and it penetrates some distance - which is how cancer treatment works; however attempting to suggest that masters in physics is related to the media is insulting, and I'd be thankful if you could refrain.
            – UKMonkey
            yesterday






          • 1




            I can vouch for the heating effect of visible light on skin. A 2kW theatrical followspot with an IR-reducing coating (so probably 100-200W in 15-20cm diameter) has a beam powerful enough that you wouldn't leave your hand in it for long or choose to put it back in. For pure visible light, even a couple of hundred mW of 532nm laser (parallel beam) certainly stings enough to deter longer-term testing
            – Chris H
            yesterday
















          17














          Assuming the intensity is the same, microwaves are more dangerous than visible light because they penetrate the skin to a greater depth (1-2 cm; more info is in Wikipedia).



          Humans have more adaptation to visible light than to microwave radiation, because they were exposed to light for millions of years. This is expressed in two ways:




          • Proteins in the epidermis (outer skin layer) are more resistant to heat than those in the deeper skin layers

          • There are more nerve endings in the outer layers of skin, so dangerous heating by visible light causes more pain, urging you to escape the dangerous situation


          Oh, and the most obvious difference: visible light is visible. Dangerous levels of visible light (e.g. in a solar cooker), to our eyes, look blinding and obviously dangerous. Dangerous levels of microwave radiation are invisible.






          share|cite|improve this answer























          • In general, anything that penetrates more is less dangerous; since it means it's less ionising. For example, gamma radiation; when it does ionise does a whole lot of damage... however it's such high energy, most of them are able to pass through the body without impacting it at all. While something like an alpha particle (sure, it's not EM, but concept applies) becomes VERY dangerous because it doesn't penetrate at all. Obviously, reflecting and passing through are the same in that the energy isn't absorbed by the organs; meaning no ionisation occurs.
            – UKMonkey
            yesterday








          • 1




            Wikipedia doesn't seem to support your claim. For instance, the microwave intensity found to cause cataracts in rabbits is 150 mW/cm2 for 100 minutes. That is roughly equal to the intensity of natural sunlight. I believe staring at the Sun will make you blind much faster.
            – Dmitry Grigoryev
            yesterday






          • 3




            @UKMonkey Re "alpha is more dangerous than gamma because it penetrates less": Simply not true: "exposure to most alpha particles originating outside the body is not a serious hazard." Or maybe a Briton would trust the BBC. The same applies to light: The skin absorbs/reflects lots of it and protects underlying tissue; while microwaves can penetrate it and reach living cells.
            – Peter A. Schneider
            yesterday












          • @PeterA.Schneider Sure - emphasis on outside the body. Inside the body is a very different story. That's because our skin has been very well adapted to protect us. The point is that you can't just say "it's more penetrating -> it's more dangerous" EM Radiation isn't a gun - if it goes through you, it's harmless. Now there's a sweet spot where all the radiation is absorbed in the body, and it penetrates some distance - which is how cancer treatment works; however attempting to suggest that masters in physics is related to the media is insulting, and I'd be thankful if you could refrain.
            – UKMonkey
            yesterday






          • 1




            I can vouch for the heating effect of visible light on skin. A 2kW theatrical followspot with an IR-reducing coating (so probably 100-200W in 15-20cm diameter) has a beam powerful enough that you wouldn't leave your hand in it for long or choose to put it back in. For pure visible light, even a couple of hundred mW of 532nm laser (parallel beam) certainly stings enough to deter longer-term testing
            – Chris H
            yesterday














          17












          17








          17






          Assuming the intensity is the same, microwaves are more dangerous than visible light because they penetrate the skin to a greater depth (1-2 cm; more info is in Wikipedia).



          Humans have more adaptation to visible light than to microwave radiation, because they were exposed to light for millions of years. This is expressed in two ways:




          • Proteins in the epidermis (outer skin layer) are more resistant to heat than those in the deeper skin layers

          • There are more nerve endings in the outer layers of skin, so dangerous heating by visible light causes more pain, urging you to escape the dangerous situation


          Oh, and the most obvious difference: visible light is visible. Dangerous levels of visible light (e.g. in a solar cooker), to our eyes, look blinding and obviously dangerous. Dangerous levels of microwave radiation are invisible.






          share|cite|improve this answer














          Assuming the intensity is the same, microwaves are more dangerous than visible light because they penetrate the skin to a greater depth (1-2 cm; more info is in Wikipedia).



          Humans have more adaptation to visible light than to microwave radiation, because they were exposed to light for millions of years. This is expressed in two ways:




          • Proteins in the epidermis (outer skin layer) are more resistant to heat than those in the deeper skin layers

          • There are more nerve endings in the outer layers of skin, so dangerous heating by visible light causes more pain, urging you to escape the dangerous situation


          Oh, and the most obvious difference: visible light is visible. Dangerous levels of visible light (e.g. in a solar cooker), to our eyes, look blinding and obviously dangerous. Dangerous levels of microwave radiation are invisible.







          share|cite|improve this answer














          share|cite|improve this answer



          share|cite|improve this answer








          edited yesterday

























          answered yesterday









          user27542user27542

          5791415




          5791415












          • In general, anything that penetrates more is less dangerous; since it means it's less ionising. For example, gamma radiation; when it does ionise does a whole lot of damage... however it's such high energy, most of them are able to pass through the body without impacting it at all. While something like an alpha particle (sure, it's not EM, but concept applies) becomes VERY dangerous because it doesn't penetrate at all. Obviously, reflecting and passing through are the same in that the energy isn't absorbed by the organs; meaning no ionisation occurs.
            – UKMonkey
            yesterday








          • 1




            Wikipedia doesn't seem to support your claim. For instance, the microwave intensity found to cause cataracts in rabbits is 150 mW/cm2 for 100 minutes. That is roughly equal to the intensity of natural sunlight. I believe staring at the Sun will make you blind much faster.
            – Dmitry Grigoryev
            yesterday






          • 3




            @UKMonkey Re "alpha is more dangerous than gamma because it penetrates less": Simply not true: "exposure to most alpha particles originating outside the body is not a serious hazard." Or maybe a Briton would trust the BBC. The same applies to light: The skin absorbs/reflects lots of it and protects underlying tissue; while microwaves can penetrate it and reach living cells.
            – Peter A. Schneider
            yesterday












          • @PeterA.Schneider Sure - emphasis on outside the body. Inside the body is a very different story. That's because our skin has been very well adapted to protect us. The point is that you can't just say "it's more penetrating -> it's more dangerous" EM Radiation isn't a gun - if it goes through you, it's harmless. Now there's a sweet spot where all the radiation is absorbed in the body, and it penetrates some distance - which is how cancer treatment works; however attempting to suggest that masters in physics is related to the media is insulting, and I'd be thankful if you could refrain.
            – UKMonkey
            yesterday






          • 1




            I can vouch for the heating effect of visible light on skin. A 2kW theatrical followspot with an IR-reducing coating (so probably 100-200W in 15-20cm diameter) has a beam powerful enough that you wouldn't leave your hand in it for long or choose to put it back in. For pure visible light, even a couple of hundred mW of 532nm laser (parallel beam) certainly stings enough to deter longer-term testing
            – Chris H
            yesterday


















          • In general, anything that penetrates more is less dangerous; since it means it's less ionising. For example, gamma radiation; when it does ionise does a whole lot of damage... however it's such high energy, most of them are able to pass through the body without impacting it at all. While something like an alpha particle (sure, it's not EM, but concept applies) becomes VERY dangerous because it doesn't penetrate at all. Obviously, reflecting and passing through are the same in that the energy isn't absorbed by the organs; meaning no ionisation occurs.
            – UKMonkey
            yesterday








          • 1




            Wikipedia doesn't seem to support your claim. For instance, the microwave intensity found to cause cataracts in rabbits is 150 mW/cm2 for 100 minutes. That is roughly equal to the intensity of natural sunlight. I believe staring at the Sun will make you blind much faster.
            – Dmitry Grigoryev
            yesterday






          • 3




            @UKMonkey Re "alpha is more dangerous than gamma because it penetrates less": Simply not true: "exposure to most alpha particles originating outside the body is not a serious hazard." Or maybe a Briton would trust the BBC. The same applies to light: The skin absorbs/reflects lots of it and protects underlying tissue; while microwaves can penetrate it and reach living cells.
            – Peter A. Schneider
            yesterday












          • @PeterA.Schneider Sure - emphasis on outside the body. Inside the body is a very different story. That's because our skin has been very well adapted to protect us. The point is that you can't just say "it's more penetrating -> it's more dangerous" EM Radiation isn't a gun - if it goes through you, it's harmless. Now there's a sweet spot where all the radiation is absorbed in the body, and it penetrates some distance - which is how cancer treatment works; however attempting to suggest that masters in physics is related to the media is insulting, and I'd be thankful if you could refrain.
            – UKMonkey
            yesterday






          • 1




            I can vouch for the heating effect of visible light on skin. A 2kW theatrical followspot with an IR-reducing coating (so probably 100-200W in 15-20cm diameter) has a beam powerful enough that you wouldn't leave your hand in it for long or choose to put it back in. For pure visible light, even a couple of hundred mW of 532nm laser (parallel beam) certainly stings enough to deter longer-term testing
            – Chris H
            yesterday
















          In general, anything that penetrates more is less dangerous; since it means it's less ionising. For example, gamma radiation; when it does ionise does a whole lot of damage... however it's such high energy, most of them are able to pass through the body without impacting it at all. While something like an alpha particle (sure, it's not EM, but concept applies) becomes VERY dangerous because it doesn't penetrate at all. Obviously, reflecting and passing through are the same in that the energy isn't absorbed by the organs; meaning no ionisation occurs.
          – UKMonkey
          yesterday






          In general, anything that penetrates more is less dangerous; since it means it's less ionising. For example, gamma radiation; when it does ionise does a whole lot of damage... however it's such high energy, most of them are able to pass through the body without impacting it at all. While something like an alpha particle (sure, it's not EM, but concept applies) becomes VERY dangerous because it doesn't penetrate at all. Obviously, reflecting and passing through are the same in that the energy isn't absorbed by the organs; meaning no ionisation occurs.
          – UKMonkey
          yesterday






          1




          1




          Wikipedia doesn't seem to support your claim. For instance, the microwave intensity found to cause cataracts in rabbits is 150 mW/cm2 for 100 minutes. That is roughly equal to the intensity of natural sunlight. I believe staring at the Sun will make you blind much faster.
          – Dmitry Grigoryev
          yesterday




          Wikipedia doesn't seem to support your claim. For instance, the microwave intensity found to cause cataracts in rabbits is 150 mW/cm2 for 100 minutes. That is roughly equal to the intensity of natural sunlight. I believe staring at the Sun will make you blind much faster.
          – Dmitry Grigoryev
          yesterday




          3




          3




          @UKMonkey Re "alpha is more dangerous than gamma because it penetrates less": Simply not true: "exposure to most alpha particles originating outside the body is not a serious hazard." Or maybe a Briton would trust the BBC. The same applies to light: The skin absorbs/reflects lots of it and protects underlying tissue; while microwaves can penetrate it and reach living cells.
          – Peter A. Schneider
          yesterday






          @UKMonkey Re "alpha is more dangerous than gamma because it penetrates less": Simply not true: "exposure to most alpha particles originating outside the body is not a serious hazard." Or maybe a Briton would trust the BBC. The same applies to light: The skin absorbs/reflects lots of it and protects underlying tissue; while microwaves can penetrate it and reach living cells.
          – Peter A. Schneider
          yesterday














          @PeterA.Schneider Sure - emphasis on outside the body. Inside the body is a very different story. That's because our skin has been very well adapted to protect us. The point is that you can't just say "it's more penetrating -> it's more dangerous" EM Radiation isn't a gun - if it goes through you, it's harmless. Now there's a sweet spot where all the radiation is absorbed in the body, and it penetrates some distance - which is how cancer treatment works; however attempting to suggest that masters in physics is related to the media is insulting, and I'd be thankful if you could refrain.
          – UKMonkey
          yesterday




          @PeterA.Schneider Sure - emphasis on outside the body. Inside the body is a very different story. That's because our skin has been very well adapted to protect us. The point is that you can't just say "it's more penetrating -> it's more dangerous" EM Radiation isn't a gun - if it goes through you, it's harmless. Now there's a sweet spot where all the radiation is absorbed in the body, and it penetrates some distance - which is how cancer treatment works; however attempting to suggest that masters in physics is related to the media is insulting, and I'd be thankful if you could refrain.
          – UKMonkey
          yesterday




          1




          1




          I can vouch for the heating effect of visible light on skin. A 2kW theatrical followspot with an IR-reducing coating (so probably 100-200W in 15-20cm diameter) has a beam powerful enough that you wouldn't leave your hand in it for long or choose to put it back in. For pure visible light, even a couple of hundred mW of 532nm laser (parallel beam) certainly stings enough to deter longer-term testing
          – Chris H
          yesterday




          I can vouch for the heating effect of visible light on skin. A 2kW theatrical followspot with an IR-reducing coating (so probably 100-200W in 15-20cm diameter) has a beam powerful enough that you wouldn't leave your hand in it for long or choose to put it back in. For pure visible light, even a couple of hundred mW of 532nm laser (parallel beam) certainly stings enough to deter longer-term testing
          – Chris H
          yesterday











          15














          The danger of electromagnetic waves is a function of the photon energy, the intensity of the source and your distance from it, and the qualitative nature of the interaction of a specific frequency with organic matter.



          That latter bit is very complex. The visible spectrum, down to about infra-red, doesn't penetrate the top layer of skin, or most clothing, so for the most part its interaction is limited to heating. Strong infrared can certainly cause burns. Strong visible light can certainly cause eye damage. But very high-intensity sources of visible light are rare in daily life, and very notably, we can see them, and avoid them.



          Further down in the energy spectrum (longer wavelength) you get the "millimeter waves" of airport scanners, which can penetrate clothing but not skin, and then you're into the microwaves, uhf and vfh radio waves, and then the radio waves called shortwaves (high frequency), medium waves and longwaves (low frequency). Microwaves can penetrate into flesh, and radio frequencies can entirely traverse a human body, and these can cause very, very severe deep-tissue burns. Certain frequencies can also interfere with cardiac rhythm, which can be as fatal as it sounds.



          Your home wifi equipment produces microwaves on pretty much the same frequencies as your microwave oven, but at milliwatt power and dispersed in all directions. The oven dumps hundreds of watts into a small enclosed space. That's the difference.



          And we live in a sea of radio waves from microwave (cell phone, wifi) though uhf and vhf (two-way radios, broadcast TV and FM) and lower (broadcast radio). The key is the power. If you grab the antenna of your uncle's 500-W ham radio when he keys the mic, or climb the tower of a multi-kilowatt tv station, you'll get hurt, maybe very badly. But going about your normal business, you're probably absorbing less than a milliwatt of radio energy. And the only effect is heating, so it's little different than being in a room that's very slightly warmer.



          Now, moving upward in energy from the visible spectrum, you get ultraviolet, x-rays and then a vast spectrum of increasing energy gamma. Not only can they penetrate into flesh, they have a very specific dirty trick: they have enough photon energy to ionize molecules, and when that happens to our dna and proteins, we start to have very bad days. This is a very specific capability that begins at a certain energy threshold.



          Microwaves, as you've remarked, are in the opposite direction from UV, X-rays and gamma: lower photon energy, longer wavelength. They cannot duplicate the ionization danger of higher energies, no matter how intense their sources are.






          share|cite|improve this answer








          New contributor




          CCTO is a new contributor to this site. Take care in asking for clarification, commenting, and answering.
          Check out our Code of Conduct.














          • 1




            Certain frequencies can also interfere with cardiac rhythm, can you elaborate on that? I'm aware of the case of pacemakers, but how would this work in the fully natural situation? If it's not resonance you're talking about, then what is it?
            – gerrit
            16 hours ago


















          15














          The danger of electromagnetic waves is a function of the photon energy, the intensity of the source and your distance from it, and the qualitative nature of the interaction of a specific frequency with organic matter.



          That latter bit is very complex. The visible spectrum, down to about infra-red, doesn't penetrate the top layer of skin, or most clothing, so for the most part its interaction is limited to heating. Strong infrared can certainly cause burns. Strong visible light can certainly cause eye damage. But very high-intensity sources of visible light are rare in daily life, and very notably, we can see them, and avoid them.



          Further down in the energy spectrum (longer wavelength) you get the "millimeter waves" of airport scanners, which can penetrate clothing but not skin, and then you're into the microwaves, uhf and vfh radio waves, and then the radio waves called shortwaves (high frequency), medium waves and longwaves (low frequency). Microwaves can penetrate into flesh, and radio frequencies can entirely traverse a human body, and these can cause very, very severe deep-tissue burns. Certain frequencies can also interfere with cardiac rhythm, which can be as fatal as it sounds.



          Your home wifi equipment produces microwaves on pretty much the same frequencies as your microwave oven, but at milliwatt power and dispersed in all directions. The oven dumps hundreds of watts into a small enclosed space. That's the difference.



          And we live in a sea of radio waves from microwave (cell phone, wifi) though uhf and vhf (two-way radios, broadcast TV and FM) and lower (broadcast radio). The key is the power. If you grab the antenna of your uncle's 500-W ham radio when he keys the mic, or climb the tower of a multi-kilowatt tv station, you'll get hurt, maybe very badly. But going about your normal business, you're probably absorbing less than a milliwatt of radio energy. And the only effect is heating, so it's little different than being in a room that's very slightly warmer.



          Now, moving upward in energy from the visible spectrum, you get ultraviolet, x-rays and then a vast spectrum of increasing energy gamma. Not only can they penetrate into flesh, they have a very specific dirty trick: they have enough photon energy to ionize molecules, and when that happens to our dna and proteins, we start to have very bad days. This is a very specific capability that begins at a certain energy threshold.



          Microwaves, as you've remarked, are in the opposite direction from UV, X-rays and gamma: lower photon energy, longer wavelength. They cannot duplicate the ionization danger of higher energies, no matter how intense their sources are.






          share|cite|improve this answer








          New contributor




          CCTO is a new contributor to this site. Take care in asking for clarification, commenting, and answering.
          Check out our Code of Conduct.














          • 1




            Certain frequencies can also interfere with cardiac rhythm, can you elaborate on that? I'm aware of the case of pacemakers, but how would this work in the fully natural situation? If it's not resonance you're talking about, then what is it?
            – gerrit
            16 hours ago
















          15












          15








          15






          The danger of electromagnetic waves is a function of the photon energy, the intensity of the source and your distance from it, and the qualitative nature of the interaction of a specific frequency with organic matter.



          That latter bit is very complex. The visible spectrum, down to about infra-red, doesn't penetrate the top layer of skin, or most clothing, so for the most part its interaction is limited to heating. Strong infrared can certainly cause burns. Strong visible light can certainly cause eye damage. But very high-intensity sources of visible light are rare in daily life, and very notably, we can see them, and avoid them.



          Further down in the energy spectrum (longer wavelength) you get the "millimeter waves" of airport scanners, which can penetrate clothing but not skin, and then you're into the microwaves, uhf and vfh radio waves, and then the radio waves called shortwaves (high frequency), medium waves and longwaves (low frequency). Microwaves can penetrate into flesh, and radio frequencies can entirely traverse a human body, and these can cause very, very severe deep-tissue burns. Certain frequencies can also interfere with cardiac rhythm, which can be as fatal as it sounds.



          Your home wifi equipment produces microwaves on pretty much the same frequencies as your microwave oven, but at milliwatt power and dispersed in all directions. The oven dumps hundreds of watts into a small enclosed space. That's the difference.



          And we live in a sea of radio waves from microwave (cell phone, wifi) though uhf and vhf (two-way radios, broadcast TV and FM) and lower (broadcast radio). The key is the power. If you grab the antenna of your uncle's 500-W ham radio when he keys the mic, or climb the tower of a multi-kilowatt tv station, you'll get hurt, maybe very badly. But going about your normal business, you're probably absorbing less than a milliwatt of radio energy. And the only effect is heating, so it's little different than being in a room that's very slightly warmer.



          Now, moving upward in energy from the visible spectrum, you get ultraviolet, x-rays and then a vast spectrum of increasing energy gamma. Not only can they penetrate into flesh, they have a very specific dirty trick: they have enough photon energy to ionize molecules, and when that happens to our dna and proteins, we start to have very bad days. This is a very specific capability that begins at a certain energy threshold.



          Microwaves, as you've remarked, are in the opposite direction from UV, X-rays and gamma: lower photon energy, longer wavelength. They cannot duplicate the ionization danger of higher energies, no matter how intense their sources are.






          share|cite|improve this answer








          New contributor




          CCTO is a new contributor to this site. Take care in asking for clarification, commenting, and answering.
          Check out our Code of Conduct.









          The danger of electromagnetic waves is a function of the photon energy, the intensity of the source and your distance from it, and the qualitative nature of the interaction of a specific frequency with organic matter.



          That latter bit is very complex. The visible spectrum, down to about infra-red, doesn't penetrate the top layer of skin, or most clothing, so for the most part its interaction is limited to heating. Strong infrared can certainly cause burns. Strong visible light can certainly cause eye damage. But very high-intensity sources of visible light are rare in daily life, and very notably, we can see them, and avoid them.



          Further down in the energy spectrum (longer wavelength) you get the "millimeter waves" of airport scanners, which can penetrate clothing but not skin, and then you're into the microwaves, uhf and vfh radio waves, and then the radio waves called shortwaves (high frequency), medium waves and longwaves (low frequency). Microwaves can penetrate into flesh, and radio frequencies can entirely traverse a human body, and these can cause very, very severe deep-tissue burns. Certain frequencies can also interfere with cardiac rhythm, which can be as fatal as it sounds.



          Your home wifi equipment produces microwaves on pretty much the same frequencies as your microwave oven, but at milliwatt power and dispersed in all directions. The oven dumps hundreds of watts into a small enclosed space. That's the difference.



          And we live in a sea of radio waves from microwave (cell phone, wifi) though uhf and vhf (two-way radios, broadcast TV and FM) and lower (broadcast radio). The key is the power. If you grab the antenna of your uncle's 500-W ham radio when he keys the mic, or climb the tower of a multi-kilowatt tv station, you'll get hurt, maybe very badly. But going about your normal business, you're probably absorbing less than a milliwatt of radio energy. And the only effect is heating, so it's little different than being in a room that's very slightly warmer.



          Now, moving upward in energy from the visible spectrum, you get ultraviolet, x-rays and then a vast spectrum of increasing energy gamma. Not only can they penetrate into flesh, they have a very specific dirty trick: they have enough photon energy to ionize molecules, and when that happens to our dna and proteins, we start to have very bad days. This is a very specific capability that begins at a certain energy threshold.



          Microwaves, as you've remarked, are in the opposite direction from UV, X-rays and gamma: lower photon energy, longer wavelength. They cannot duplicate the ionization danger of higher energies, no matter how intense their sources are.







          share|cite|improve this answer








          New contributor




          CCTO is a new contributor to this site. Take care in asking for clarification, commenting, and answering.
          Check out our Code of Conduct.









          share|cite|improve this answer



          share|cite|improve this answer






          New contributor




          CCTO is a new contributor to this site. Take care in asking for clarification, commenting, and answering.
          Check out our Code of Conduct.









          answered yesterday









          CCTOCCTO

          2513




          2513




          New contributor




          CCTO is a new contributor to this site. Take care in asking for clarification, commenting, and answering.
          Check out our Code of Conduct.





          New contributor





          CCTO is a new contributor to this site. Take care in asking for clarification, commenting, and answering.
          Check out our Code of Conduct.






          CCTO is a new contributor to this site. Take care in asking for clarification, commenting, and answering.
          Check out our Code of Conduct.








          • 1




            Certain frequencies can also interfere with cardiac rhythm, can you elaborate on that? I'm aware of the case of pacemakers, but how would this work in the fully natural situation? If it's not resonance you're talking about, then what is it?
            – gerrit
            16 hours ago
















          • 1




            Certain frequencies can also interfere with cardiac rhythm, can you elaborate on that? I'm aware of the case of pacemakers, but how would this work in the fully natural situation? If it's not resonance you're talking about, then what is it?
            – gerrit
            16 hours ago










          1




          1




          Certain frequencies can also interfere with cardiac rhythm, can you elaborate on that? I'm aware of the case of pacemakers, but how would this work in the fully natural situation? If it's not resonance you're talking about, then what is it?
          – gerrit
          16 hours ago






          Certain frequencies can also interfere with cardiac rhythm, can you elaborate on that? I'm aware of the case of pacemakers, but how would this work in the fully natural situation? If it's not resonance you're talking about, then what is it?
          – gerrit
          16 hours ago













          8














          Other answers already point out the matter of intensity. If you have a 1kW microwave that cooks your chicken and so call microwaves dangerous, you can equally cook chicken with 1kW visible light bulb. The difference is mostly how well and how deep the absorption goes, but the amount of energy is the same if the absorbed power is the same.



          What makes the difference:




          • Absorption coefficient; how WELL the light is absorbed. So, a black chicken will cook well in strong visible light, but a white chicken will require higher power, because it reflects more. Microwaves work well because they penetrate deeper before they get absorbed (due to longer wavelength), but also absorb WELL because the frequency is tuned near the resonance for water molecules.

          • Resonances; If the wavelength matches exactly to one of the transition frequencies for molecules/atoms in the target, most of the energy is selectively absorbed just by those molecules. So, if you tuned your light specifically to a transition that breaks some specific bonds, or heat just specific tissues. This can do more damage because it may change chemistry, but luckily, breaking bonds requires quite high frequencies - see next case below. With MW and IR, you will still end up just heating the sample if you find a resonance (resonances in MW, IR and visible are mostly vibrational and rotational transitions, not bond changing, except for red-colored substances reacting to visible light, which you notice when red dyes bleach quickly in intense light).

          • Ionization; If the energy of a SINGLE photon is enough to kick off an electron from a molecule/atom, then it's dangerous because it's actively affecting the chemistry (note that this is similar result than the resonant case above, but instead of having a precise frequency, it has way too much energy, with similar results). This is what is called ionising radiation (gamma/X-rays, down to UV range).


          Note that resonance just means good absorbtion, nothing mystical. Water is mostly transparent for visible light because no significant vibrations of water molecule fall in this range - most are in IR and microwave, and there is another absorption range in UV.



          Rule of thumb: microwaves, IR and visible light just heat you up. It's the heat is sufficient to raise the temperature into danger zone, it's dangerous, otherwise it's harmless. Only intensity matters (Watts per square meter), not the frequency.
          Ionizing radiation (UV/X-ray/gamma) are dangerous because of chemical damage even at low intensity.



          Microwaves are NOT ionizing radiation, so the wireless and mobile signals do absolutely nothing - the power is way too low, otherwise you'd need to charge your phone every 5 minutes.






          share|cite|improve this answer


























            8














            Other answers already point out the matter of intensity. If you have a 1kW microwave that cooks your chicken and so call microwaves dangerous, you can equally cook chicken with 1kW visible light bulb. The difference is mostly how well and how deep the absorption goes, but the amount of energy is the same if the absorbed power is the same.



            What makes the difference:




            • Absorption coefficient; how WELL the light is absorbed. So, a black chicken will cook well in strong visible light, but a white chicken will require higher power, because it reflects more. Microwaves work well because they penetrate deeper before they get absorbed (due to longer wavelength), but also absorb WELL because the frequency is tuned near the resonance for water molecules.

            • Resonances; If the wavelength matches exactly to one of the transition frequencies for molecules/atoms in the target, most of the energy is selectively absorbed just by those molecules. So, if you tuned your light specifically to a transition that breaks some specific bonds, or heat just specific tissues. This can do more damage because it may change chemistry, but luckily, breaking bonds requires quite high frequencies - see next case below. With MW and IR, you will still end up just heating the sample if you find a resonance (resonances in MW, IR and visible are mostly vibrational and rotational transitions, not bond changing, except for red-colored substances reacting to visible light, which you notice when red dyes bleach quickly in intense light).

            • Ionization; If the energy of a SINGLE photon is enough to kick off an electron from a molecule/atom, then it's dangerous because it's actively affecting the chemistry (note that this is similar result than the resonant case above, but instead of having a precise frequency, it has way too much energy, with similar results). This is what is called ionising radiation (gamma/X-rays, down to UV range).


            Note that resonance just means good absorbtion, nothing mystical. Water is mostly transparent for visible light because no significant vibrations of water molecule fall in this range - most are in IR and microwave, and there is another absorption range in UV.



            Rule of thumb: microwaves, IR and visible light just heat you up. It's the heat is sufficient to raise the temperature into danger zone, it's dangerous, otherwise it's harmless. Only intensity matters (Watts per square meter), not the frequency.
            Ionizing radiation (UV/X-ray/gamma) are dangerous because of chemical damage even at low intensity.



            Microwaves are NOT ionizing radiation, so the wireless and mobile signals do absolutely nothing - the power is way too low, otherwise you'd need to charge your phone every 5 minutes.






            share|cite|improve this answer
























              8












              8








              8






              Other answers already point out the matter of intensity. If you have a 1kW microwave that cooks your chicken and so call microwaves dangerous, you can equally cook chicken with 1kW visible light bulb. The difference is mostly how well and how deep the absorption goes, but the amount of energy is the same if the absorbed power is the same.



              What makes the difference:




              • Absorption coefficient; how WELL the light is absorbed. So, a black chicken will cook well in strong visible light, but a white chicken will require higher power, because it reflects more. Microwaves work well because they penetrate deeper before they get absorbed (due to longer wavelength), but also absorb WELL because the frequency is tuned near the resonance for water molecules.

              • Resonances; If the wavelength matches exactly to one of the transition frequencies for molecules/atoms in the target, most of the energy is selectively absorbed just by those molecules. So, if you tuned your light specifically to a transition that breaks some specific bonds, or heat just specific tissues. This can do more damage because it may change chemistry, but luckily, breaking bonds requires quite high frequencies - see next case below. With MW and IR, you will still end up just heating the sample if you find a resonance (resonances in MW, IR and visible are mostly vibrational and rotational transitions, not bond changing, except for red-colored substances reacting to visible light, which you notice when red dyes bleach quickly in intense light).

              • Ionization; If the energy of a SINGLE photon is enough to kick off an electron from a molecule/atom, then it's dangerous because it's actively affecting the chemistry (note that this is similar result than the resonant case above, but instead of having a precise frequency, it has way too much energy, with similar results). This is what is called ionising radiation (gamma/X-rays, down to UV range).


              Note that resonance just means good absorbtion, nothing mystical. Water is mostly transparent for visible light because no significant vibrations of water molecule fall in this range - most are in IR and microwave, and there is another absorption range in UV.



              Rule of thumb: microwaves, IR and visible light just heat you up. It's the heat is sufficient to raise the temperature into danger zone, it's dangerous, otherwise it's harmless. Only intensity matters (Watts per square meter), not the frequency.
              Ionizing radiation (UV/X-ray/gamma) are dangerous because of chemical damage even at low intensity.



              Microwaves are NOT ionizing radiation, so the wireless and mobile signals do absolutely nothing - the power is way too low, otherwise you'd need to charge your phone every 5 minutes.






              share|cite|improve this answer












              Other answers already point out the matter of intensity. If you have a 1kW microwave that cooks your chicken and so call microwaves dangerous, you can equally cook chicken with 1kW visible light bulb. The difference is mostly how well and how deep the absorption goes, but the amount of energy is the same if the absorbed power is the same.



              What makes the difference:




              • Absorption coefficient; how WELL the light is absorbed. So, a black chicken will cook well in strong visible light, but a white chicken will require higher power, because it reflects more. Microwaves work well because they penetrate deeper before they get absorbed (due to longer wavelength), but also absorb WELL because the frequency is tuned near the resonance for water molecules.

              • Resonances; If the wavelength matches exactly to one of the transition frequencies for molecules/atoms in the target, most of the energy is selectively absorbed just by those molecules. So, if you tuned your light specifically to a transition that breaks some specific bonds, or heat just specific tissues. This can do more damage because it may change chemistry, but luckily, breaking bonds requires quite high frequencies - see next case below. With MW and IR, you will still end up just heating the sample if you find a resonance (resonances in MW, IR and visible are mostly vibrational and rotational transitions, not bond changing, except for red-colored substances reacting to visible light, which you notice when red dyes bleach quickly in intense light).

              • Ionization; If the energy of a SINGLE photon is enough to kick off an electron from a molecule/atom, then it's dangerous because it's actively affecting the chemistry (note that this is similar result than the resonant case above, but instead of having a precise frequency, it has way too much energy, with similar results). This is what is called ionising radiation (gamma/X-rays, down to UV range).


              Note that resonance just means good absorbtion, nothing mystical. Water is mostly transparent for visible light because no significant vibrations of water molecule fall in this range - most are in IR and microwave, and there is another absorption range in UV.



              Rule of thumb: microwaves, IR and visible light just heat you up. It's the heat is sufficient to raise the temperature into danger zone, it's dangerous, otherwise it's harmless. Only intensity matters (Watts per square meter), not the frequency.
              Ionizing radiation (UV/X-ray/gamma) are dangerous because of chemical damage even at low intensity.



              Microwaves are NOT ionizing radiation, so the wireless and mobile signals do absolutely nothing - the power is way too low, otherwise you'd need to charge your phone every 5 minutes.







              share|cite|improve this answer












              share|cite|improve this answer



              share|cite|improve this answer










              answered yesterday









              orionorion

              5,7281425




              5,7281425























                  5














                  It's more accurate to say microwave ovens are dangerous. Then again, so is visible light.



                  It's a question not of photon energy, but total energy. A typical microwave outputs on the order of 1 kilowatt of electromagnetic radiation which is almost entirely absorbed by the food within.



                  By comparison, the solar power at earth's surface, at maximum, is around 1 kilowatt per square meter. If it's cloudy, not at the equator, or not noon, it will be less. Most foods have a surface area of much less than a square meter, so the total electromagnetic radiation power received by something sitting in the sun is much less than a microwave oven.



                  For a fair comparison, what do you think would happen if a magnifying glass with an area of one square meter, on a very sunny day, were used to focus light on to something the size of what you'd put into a microwave oven?



                  There are a few more subtle differences. For example dangerous powers of visible light are so bright you'll surely close your eyes. Furthermore, visible light penetrates less deeply, so you're likely to feel the heat and move away before it does any more than superficial damage to your skin, like a sunburn. On the other hand, microwave radiation is invisible and penetrates more deeply, so you may suffer irreversible injury before even noticing the hazard. The cornea is especially prone to microwave injury since there's no protective reflex to protect it, it has little thermal mass and thus heats quickly, and there's little blood flow to cool it.






                  share|cite|improve this answer























                  • Of course, solar ovens are a thing - and indeed, to get a decent oven, a square meter of mirrors concentrating the sunlight to the center (with the food item) is more than enough.
                    – Luaan
                    20 hours ago
















                  5














                  It's more accurate to say microwave ovens are dangerous. Then again, so is visible light.



                  It's a question not of photon energy, but total energy. A typical microwave outputs on the order of 1 kilowatt of electromagnetic radiation which is almost entirely absorbed by the food within.



                  By comparison, the solar power at earth's surface, at maximum, is around 1 kilowatt per square meter. If it's cloudy, not at the equator, or not noon, it will be less. Most foods have a surface area of much less than a square meter, so the total electromagnetic radiation power received by something sitting in the sun is much less than a microwave oven.



                  For a fair comparison, what do you think would happen if a magnifying glass with an area of one square meter, on a very sunny day, were used to focus light on to something the size of what you'd put into a microwave oven?



                  There are a few more subtle differences. For example dangerous powers of visible light are so bright you'll surely close your eyes. Furthermore, visible light penetrates less deeply, so you're likely to feel the heat and move away before it does any more than superficial damage to your skin, like a sunburn. On the other hand, microwave radiation is invisible and penetrates more deeply, so you may suffer irreversible injury before even noticing the hazard. The cornea is especially prone to microwave injury since there's no protective reflex to protect it, it has little thermal mass and thus heats quickly, and there's little blood flow to cool it.






                  share|cite|improve this answer























                  • Of course, solar ovens are a thing - and indeed, to get a decent oven, a square meter of mirrors concentrating the sunlight to the center (with the food item) is more than enough.
                    – Luaan
                    20 hours ago














                  5












                  5








                  5






                  It's more accurate to say microwave ovens are dangerous. Then again, so is visible light.



                  It's a question not of photon energy, but total energy. A typical microwave outputs on the order of 1 kilowatt of electromagnetic radiation which is almost entirely absorbed by the food within.



                  By comparison, the solar power at earth's surface, at maximum, is around 1 kilowatt per square meter. If it's cloudy, not at the equator, or not noon, it will be less. Most foods have a surface area of much less than a square meter, so the total electromagnetic radiation power received by something sitting in the sun is much less than a microwave oven.



                  For a fair comparison, what do you think would happen if a magnifying glass with an area of one square meter, on a very sunny day, were used to focus light on to something the size of what you'd put into a microwave oven?



                  There are a few more subtle differences. For example dangerous powers of visible light are so bright you'll surely close your eyes. Furthermore, visible light penetrates less deeply, so you're likely to feel the heat and move away before it does any more than superficial damage to your skin, like a sunburn. On the other hand, microwave radiation is invisible and penetrates more deeply, so you may suffer irreversible injury before even noticing the hazard. The cornea is especially prone to microwave injury since there's no protective reflex to protect it, it has little thermal mass and thus heats quickly, and there's little blood flow to cool it.






                  share|cite|improve this answer














                  It's more accurate to say microwave ovens are dangerous. Then again, so is visible light.



                  It's a question not of photon energy, but total energy. A typical microwave outputs on the order of 1 kilowatt of electromagnetic radiation which is almost entirely absorbed by the food within.



                  By comparison, the solar power at earth's surface, at maximum, is around 1 kilowatt per square meter. If it's cloudy, not at the equator, or not noon, it will be less. Most foods have a surface area of much less than a square meter, so the total electromagnetic radiation power received by something sitting in the sun is much less than a microwave oven.



                  For a fair comparison, what do you think would happen if a magnifying glass with an area of one square meter, on a very sunny day, were used to focus light on to something the size of what you'd put into a microwave oven?



                  There are a few more subtle differences. For example dangerous powers of visible light are so bright you'll surely close your eyes. Furthermore, visible light penetrates less deeply, so you're likely to feel the heat and move away before it does any more than superficial damage to your skin, like a sunburn. On the other hand, microwave radiation is invisible and penetrates more deeply, so you may suffer irreversible injury before even noticing the hazard. The cornea is especially prone to microwave injury since there's no protective reflex to protect it, it has little thermal mass and thus heats quickly, and there's little blood flow to cool it.







                  share|cite|improve this answer














                  share|cite|improve this answer



                  share|cite|improve this answer








                  edited 17 hours ago

























                  answered yesterday









                  Phil FrostPhil Frost

                  3,39421726




                  3,39421726












                  • Of course, solar ovens are a thing - and indeed, to get a decent oven, a square meter of mirrors concentrating the sunlight to the center (with the food item) is more than enough.
                    – Luaan
                    20 hours ago


















                  • Of course, solar ovens are a thing - and indeed, to get a decent oven, a square meter of mirrors concentrating the sunlight to the center (with the food item) is more than enough.
                    – Luaan
                    20 hours ago
















                  Of course, solar ovens are a thing - and indeed, to get a decent oven, a square meter of mirrors concentrating the sunlight to the center (with the food item) is more than enough.
                  – Luaan
                  20 hours ago




                  Of course, solar ovens are a thing - and indeed, to get a decent oven, a square meter of mirrors concentrating the sunlight to the center (with the food item) is more than enough.
                  – Luaan
                  20 hours ago











                  1














                  There is a saying that "The dose makes the toxin."



                  Oxygen is the substance you most need a constant supply of. You will die after just a few minutes without oxygen.



                  But oxygen toxicity is real. Too much oxygen can harm or kill you. In fact, for billions of years all the organisms on Earth had no use for oxygen. When oxygen concentration increased in the atmosphere most life forms on Earth died off. Only the ones that adapted fast enough to tolerate and even to depend on oxygen could survive in the more oxygen rich atmosphere.



                  The same goes for every other necessary substance or environmental factor. And the same goes for every other dangerous substance or environmental factor. In high enough doses, even the most necessary things are deadly. In low enough doses, even the most deadly things can be harmless and maybe even useful.



                  Since visible light and all other frequencies of electromagnetic radiation are environmental factors, the preceding is also true for them. Too much of any frequencies of electromagnetic radiation, even the most beneficial, can be harmful or deadly, and small enough exposure to even the deadliest frequencies of electromagnetic radiation, such as X-Rays or gamma rays, can be harmless or even beneficial.



                  I remember a story by Arthur C. Clarke in which a character criticized the way that death rays in science fiction were visible to the human eye, saying that if visible light was deadly, humans couldn't live. But humans have evolved to survive the concentrations of visible light that are common on Earth. A human exposed to a concentration of visible light that was a thousand times, or a million times, or a billion times, stronger could be killed, cooked, or even instantly vaporized.



                  I also remember two other stories by Arthur C. Clarke, perhaps even in the same collection, where humans found plausible ways to create death rays out of visible light using the primitive technology of the 1950s and 1960s.



                  Some forms of radiation therapy for cancer involve using beams of X-Rays, Gamma rays, or charged particles to help kill cancer cells. So people undergoing radiation therapy are often benefited by being struck by death rays designed to kill living tissues, because the death rays are aimed at living tissues that would kill their host bodies eventually.



                  As we all know, antimatter is the most dangerous substance imagined by physicists. If a normal particle collides with its opposite antiparticle, both are annihilated and radiation is emitted.



                  You may have heard of people having PET scans for medical diagnosis. PET stands for Positron Emission Tomography. A positron is an anti electron, and thus an antiparticle. So people who had PET scans have survived, and perhaps benefited from, having minute amounts of antiparticles in their bodies.



                  So even with something as supremely deadly as antimatter, the dose makes the toxin.



                  A microwave oven does to food something very similar to what camp fires and stoves do to food, and requires about the same amount of energy per meal. The amount of energy received per second of being microwaved is many times the amount of energy per second in natural or artificial light for illumination. much



                  So being exposed to the same energy in visible light frequencies as an open fire or a stove imparts to a meal is not likely to be much better for someone than being microwaved in a microwave oven would be.






                  share|cite|improve this answer








                  New contributor




                  M.A. Golding is a new contributor to this site. Take care in asking for clarification, commenting, and answering.
                  Check out our Code of Conduct.























                    1














                    There is a saying that "The dose makes the toxin."



                    Oxygen is the substance you most need a constant supply of. You will die after just a few minutes without oxygen.



                    But oxygen toxicity is real. Too much oxygen can harm or kill you. In fact, for billions of years all the organisms on Earth had no use for oxygen. When oxygen concentration increased in the atmosphere most life forms on Earth died off. Only the ones that adapted fast enough to tolerate and even to depend on oxygen could survive in the more oxygen rich atmosphere.



                    The same goes for every other necessary substance or environmental factor. And the same goes for every other dangerous substance or environmental factor. In high enough doses, even the most necessary things are deadly. In low enough doses, even the most deadly things can be harmless and maybe even useful.



                    Since visible light and all other frequencies of electromagnetic radiation are environmental factors, the preceding is also true for them. Too much of any frequencies of electromagnetic radiation, even the most beneficial, can be harmful or deadly, and small enough exposure to even the deadliest frequencies of electromagnetic radiation, such as X-Rays or gamma rays, can be harmless or even beneficial.



                    I remember a story by Arthur C. Clarke in which a character criticized the way that death rays in science fiction were visible to the human eye, saying that if visible light was deadly, humans couldn't live. But humans have evolved to survive the concentrations of visible light that are common on Earth. A human exposed to a concentration of visible light that was a thousand times, or a million times, or a billion times, stronger could be killed, cooked, or even instantly vaporized.



                    I also remember two other stories by Arthur C. Clarke, perhaps even in the same collection, where humans found plausible ways to create death rays out of visible light using the primitive technology of the 1950s and 1960s.



                    Some forms of radiation therapy for cancer involve using beams of X-Rays, Gamma rays, or charged particles to help kill cancer cells. So people undergoing radiation therapy are often benefited by being struck by death rays designed to kill living tissues, because the death rays are aimed at living tissues that would kill their host bodies eventually.



                    As we all know, antimatter is the most dangerous substance imagined by physicists. If a normal particle collides with its opposite antiparticle, both are annihilated and radiation is emitted.



                    You may have heard of people having PET scans for medical diagnosis. PET stands for Positron Emission Tomography. A positron is an anti electron, and thus an antiparticle. So people who had PET scans have survived, and perhaps benefited from, having minute amounts of antiparticles in their bodies.



                    So even with something as supremely deadly as antimatter, the dose makes the toxin.



                    A microwave oven does to food something very similar to what camp fires and stoves do to food, and requires about the same amount of energy per meal. The amount of energy received per second of being microwaved is many times the amount of energy per second in natural or artificial light for illumination. much



                    So being exposed to the same energy in visible light frequencies as an open fire or a stove imparts to a meal is not likely to be much better for someone than being microwaved in a microwave oven would be.






                    share|cite|improve this answer








                    New contributor




                    M.A. Golding is a new contributor to this site. Take care in asking for clarification, commenting, and answering.
                    Check out our Code of Conduct.





















                      1












                      1








                      1






                      There is a saying that "The dose makes the toxin."



                      Oxygen is the substance you most need a constant supply of. You will die after just a few minutes without oxygen.



                      But oxygen toxicity is real. Too much oxygen can harm or kill you. In fact, for billions of years all the organisms on Earth had no use for oxygen. When oxygen concentration increased in the atmosphere most life forms on Earth died off. Only the ones that adapted fast enough to tolerate and even to depend on oxygen could survive in the more oxygen rich atmosphere.



                      The same goes for every other necessary substance or environmental factor. And the same goes for every other dangerous substance or environmental factor. In high enough doses, even the most necessary things are deadly. In low enough doses, even the most deadly things can be harmless and maybe even useful.



                      Since visible light and all other frequencies of electromagnetic radiation are environmental factors, the preceding is also true for them. Too much of any frequencies of electromagnetic radiation, even the most beneficial, can be harmful or deadly, and small enough exposure to even the deadliest frequencies of electromagnetic radiation, such as X-Rays or gamma rays, can be harmless or even beneficial.



                      I remember a story by Arthur C. Clarke in which a character criticized the way that death rays in science fiction were visible to the human eye, saying that if visible light was deadly, humans couldn't live. But humans have evolved to survive the concentrations of visible light that are common on Earth. A human exposed to a concentration of visible light that was a thousand times, or a million times, or a billion times, stronger could be killed, cooked, or even instantly vaporized.



                      I also remember two other stories by Arthur C. Clarke, perhaps even in the same collection, where humans found plausible ways to create death rays out of visible light using the primitive technology of the 1950s and 1960s.



                      Some forms of radiation therapy for cancer involve using beams of X-Rays, Gamma rays, or charged particles to help kill cancer cells. So people undergoing radiation therapy are often benefited by being struck by death rays designed to kill living tissues, because the death rays are aimed at living tissues that would kill their host bodies eventually.



                      As we all know, antimatter is the most dangerous substance imagined by physicists. If a normal particle collides with its opposite antiparticle, both are annihilated and radiation is emitted.



                      You may have heard of people having PET scans for medical diagnosis. PET stands for Positron Emission Tomography. A positron is an anti electron, and thus an antiparticle. So people who had PET scans have survived, and perhaps benefited from, having minute amounts of antiparticles in their bodies.



                      So even with something as supremely deadly as antimatter, the dose makes the toxin.



                      A microwave oven does to food something very similar to what camp fires and stoves do to food, and requires about the same amount of energy per meal. The amount of energy received per second of being microwaved is many times the amount of energy per second in natural or artificial light for illumination. much



                      So being exposed to the same energy in visible light frequencies as an open fire or a stove imparts to a meal is not likely to be much better for someone than being microwaved in a microwave oven would be.






                      share|cite|improve this answer








                      New contributor




                      M.A. Golding is a new contributor to this site. Take care in asking for clarification, commenting, and answering.
                      Check out our Code of Conduct.









                      There is a saying that "The dose makes the toxin."



                      Oxygen is the substance you most need a constant supply of. You will die after just a few minutes without oxygen.



                      But oxygen toxicity is real. Too much oxygen can harm or kill you. In fact, for billions of years all the organisms on Earth had no use for oxygen. When oxygen concentration increased in the atmosphere most life forms on Earth died off. Only the ones that adapted fast enough to tolerate and even to depend on oxygen could survive in the more oxygen rich atmosphere.



                      The same goes for every other necessary substance or environmental factor. And the same goes for every other dangerous substance or environmental factor. In high enough doses, even the most necessary things are deadly. In low enough doses, even the most deadly things can be harmless and maybe even useful.



                      Since visible light and all other frequencies of electromagnetic radiation are environmental factors, the preceding is also true for them. Too much of any frequencies of electromagnetic radiation, even the most beneficial, can be harmful or deadly, and small enough exposure to even the deadliest frequencies of electromagnetic radiation, such as X-Rays or gamma rays, can be harmless or even beneficial.



                      I remember a story by Arthur C. Clarke in which a character criticized the way that death rays in science fiction were visible to the human eye, saying that if visible light was deadly, humans couldn't live. But humans have evolved to survive the concentrations of visible light that are common on Earth. A human exposed to a concentration of visible light that was a thousand times, or a million times, or a billion times, stronger could be killed, cooked, or even instantly vaporized.



                      I also remember two other stories by Arthur C. Clarke, perhaps even in the same collection, where humans found plausible ways to create death rays out of visible light using the primitive technology of the 1950s and 1960s.



                      Some forms of radiation therapy for cancer involve using beams of X-Rays, Gamma rays, or charged particles to help kill cancer cells. So people undergoing radiation therapy are often benefited by being struck by death rays designed to kill living tissues, because the death rays are aimed at living tissues that would kill their host bodies eventually.



                      As we all know, antimatter is the most dangerous substance imagined by physicists. If a normal particle collides with its opposite antiparticle, both are annihilated and radiation is emitted.



                      You may have heard of people having PET scans for medical diagnosis. PET stands for Positron Emission Tomography. A positron is an anti electron, and thus an antiparticle. So people who had PET scans have survived, and perhaps benefited from, having minute amounts of antiparticles in their bodies.



                      So even with something as supremely deadly as antimatter, the dose makes the toxin.



                      A microwave oven does to food something very similar to what camp fires and stoves do to food, and requires about the same amount of energy per meal. The amount of energy received per second of being microwaved is many times the amount of energy per second in natural or artificial light for illumination. much



                      So being exposed to the same energy in visible light frequencies as an open fire or a stove imparts to a meal is not likely to be much better for someone than being microwaved in a microwave oven would be.







                      share|cite|improve this answer








                      New contributor




                      M.A. Golding is a new contributor to this site. Take care in asking for clarification, commenting, and answering.
                      Check out our Code of Conduct.









                      share|cite|improve this answer



                      share|cite|improve this answer






                      New contributor




                      M.A. Golding is a new contributor to this site. Take care in asking for clarification, commenting, and answering.
                      Check out our Code of Conduct.









                      answered 8 hours ago









                      M.A. GoldingM.A. Golding

                      111




                      111




                      New contributor




                      M.A. Golding is a new contributor to this site. Take care in asking for clarification, commenting, and answering.
                      Check out our Code of Conduct.





                      New contributor





                      M.A. Golding is a new contributor to this site. Take care in asking for clarification, commenting, and answering.
                      Check out our Code of Conduct.






                      M.A. Golding is a new contributor to this site. Take care in asking for clarification, commenting, and answering.
                      Check out our Code of Conduct.























                          0














                          When we say that microwaves are less energetic, we are talking about the energy in a single photon. The number of photons is also important.



                          A single microwave photon is utterly harmless. Its only effect is heat, and it takes a serious amount of heat to damage us. But enough heat, in any form, will kill.



                          Visible light has enough energy that single photons can cause chemical reactions, but only in sensitive compounds. That is what happens in our eyes. The chemicals in our eyes are carefully constructed to be extra sensitive to light and that is what makes light visible.



                          Ultraviolet light is worse. Here the photons carry enough energy to cause unwanted chemical reactions in most organic compounds. Sun burn and skin cancer results.



                          Gamma ray photons from radioactivity are even worse, but they are fortunately rare.






                          share|cite|improve this answer


























                            0














                            When we say that microwaves are less energetic, we are talking about the energy in a single photon. The number of photons is also important.



                            A single microwave photon is utterly harmless. Its only effect is heat, and it takes a serious amount of heat to damage us. But enough heat, in any form, will kill.



                            Visible light has enough energy that single photons can cause chemical reactions, but only in sensitive compounds. That is what happens in our eyes. The chemicals in our eyes are carefully constructed to be extra sensitive to light and that is what makes light visible.



                            Ultraviolet light is worse. Here the photons carry enough energy to cause unwanted chemical reactions in most organic compounds. Sun burn and skin cancer results.



                            Gamma ray photons from radioactivity are even worse, but they are fortunately rare.






                            share|cite|improve this answer
























                              0












                              0








                              0






                              When we say that microwaves are less energetic, we are talking about the energy in a single photon. The number of photons is also important.



                              A single microwave photon is utterly harmless. Its only effect is heat, and it takes a serious amount of heat to damage us. But enough heat, in any form, will kill.



                              Visible light has enough energy that single photons can cause chemical reactions, but only in sensitive compounds. That is what happens in our eyes. The chemicals in our eyes are carefully constructed to be extra sensitive to light and that is what makes light visible.



                              Ultraviolet light is worse. Here the photons carry enough energy to cause unwanted chemical reactions in most organic compounds. Sun burn and skin cancer results.



                              Gamma ray photons from radioactivity are even worse, but they are fortunately rare.






                              share|cite|improve this answer












                              When we say that microwaves are less energetic, we are talking about the energy in a single photon. The number of photons is also important.



                              A single microwave photon is utterly harmless. Its only effect is heat, and it takes a serious amount of heat to damage us. But enough heat, in any form, will kill.



                              Visible light has enough energy that single photons can cause chemical reactions, but only in sensitive compounds. That is what happens in our eyes. The chemicals in our eyes are carefully constructed to be extra sensitive to light and that is what makes light visible.



                              Ultraviolet light is worse. Here the photons carry enough energy to cause unwanted chemical reactions in most organic compounds. Sun burn and skin cancer results.



                              Gamma ray photons from radioactivity are even worse, but they are fortunately rare.







                              share|cite|improve this answer












                              share|cite|improve this answer



                              share|cite|improve this answer










                              answered yesterday









                              Stig HemmerStig Hemmer

                              21924




                              21924























                                  0














                                  There is nothing inherently more or less dangerous about microwaves. Yes, the type of damage via microwave vs. visible light vs. xray are different but weather or not the light cause damage has the same factor for all the spectrum of light - intensity.



                                  Remember, modern computer-using humans constantly volunteer to be bather in microwaves. Wifi uses exactly the same frequency as microwave ovens. The difference between wifi and microwave ovens is the wattage - the amount of power used to generate the light - the intensity.



                                  You can cook with visible light if you pump enough power into it - or somehow concentrate it. This is how sun ovens work and how you can burn paper with a magnifying glass. You can also cook with xrays if it is intense enough.



                                  Side note: Most people don't realise this but high intensity infrared light can blind you as surely as staring at the sun. Just because it is invisible to your eyes does not mean the photons does not hit your retina. High intensity infrared spotlights are sold as part of security systems for infrared sensitive cameras (night vision).






                                  share|cite|improve this answer





















                                  • "High intensity" is relative, of course; most spotlights used for "night vision cameras" are still just a few watts, which is rather small compared to sunlight. There is a danger mainly because you don't realize you're too close to the light (and there's no eye response to the increasing amount of light hitting your eyes), and if it's dark, your pupils are maximally dilated. You don't see microwaves, but above about 20 W, the heat is quite noticeable, and a 200 W light is unmistakably warm (those are usually used for heating, though, not IR camera illumination).
                                    – Luaan
                                    20 hours ago
















                                  0














                                  There is nothing inherently more or less dangerous about microwaves. Yes, the type of damage via microwave vs. visible light vs. xray are different but weather or not the light cause damage has the same factor for all the spectrum of light - intensity.



                                  Remember, modern computer-using humans constantly volunteer to be bather in microwaves. Wifi uses exactly the same frequency as microwave ovens. The difference between wifi and microwave ovens is the wattage - the amount of power used to generate the light - the intensity.



                                  You can cook with visible light if you pump enough power into it - or somehow concentrate it. This is how sun ovens work and how you can burn paper with a magnifying glass. You can also cook with xrays if it is intense enough.



                                  Side note: Most people don't realise this but high intensity infrared light can blind you as surely as staring at the sun. Just because it is invisible to your eyes does not mean the photons does not hit your retina. High intensity infrared spotlights are sold as part of security systems for infrared sensitive cameras (night vision).






                                  share|cite|improve this answer





















                                  • "High intensity" is relative, of course; most spotlights used for "night vision cameras" are still just a few watts, which is rather small compared to sunlight. There is a danger mainly because you don't realize you're too close to the light (and there's no eye response to the increasing amount of light hitting your eyes), and if it's dark, your pupils are maximally dilated. You don't see microwaves, but above about 20 W, the heat is quite noticeable, and a 200 W light is unmistakably warm (those are usually used for heating, though, not IR camera illumination).
                                    – Luaan
                                    20 hours ago














                                  0












                                  0








                                  0






                                  There is nothing inherently more or less dangerous about microwaves. Yes, the type of damage via microwave vs. visible light vs. xray are different but weather or not the light cause damage has the same factor for all the spectrum of light - intensity.



                                  Remember, modern computer-using humans constantly volunteer to be bather in microwaves. Wifi uses exactly the same frequency as microwave ovens. The difference between wifi and microwave ovens is the wattage - the amount of power used to generate the light - the intensity.



                                  You can cook with visible light if you pump enough power into it - or somehow concentrate it. This is how sun ovens work and how you can burn paper with a magnifying glass. You can also cook with xrays if it is intense enough.



                                  Side note: Most people don't realise this but high intensity infrared light can blind you as surely as staring at the sun. Just because it is invisible to your eyes does not mean the photons does not hit your retina. High intensity infrared spotlights are sold as part of security systems for infrared sensitive cameras (night vision).






                                  share|cite|improve this answer












                                  There is nothing inherently more or less dangerous about microwaves. Yes, the type of damage via microwave vs. visible light vs. xray are different but weather or not the light cause damage has the same factor for all the spectrum of light - intensity.



                                  Remember, modern computer-using humans constantly volunteer to be bather in microwaves. Wifi uses exactly the same frequency as microwave ovens. The difference between wifi and microwave ovens is the wattage - the amount of power used to generate the light - the intensity.



                                  You can cook with visible light if you pump enough power into it - or somehow concentrate it. This is how sun ovens work and how you can burn paper with a magnifying glass. You can also cook with xrays if it is intense enough.



                                  Side note: Most people don't realise this but high intensity infrared light can blind you as surely as staring at the sun. Just because it is invisible to your eyes does not mean the photons does not hit your retina. High intensity infrared spotlights are sold as part of security systems for infrared sensitive cameras (night vision).







                                  share|cite|improve this answer












                                  share|cite|improve this answer



                                  share|cite|improve this answer










                                  answered yesterday









                                  slebetmanslebetman

                                  21026




                                  21026












                                  • "High intensity" is relative, of course; most spotlights used for "night vision cameras" are still just a few watts, which is rather small compared to sunlight. There is a danger mainly because you don't realize you're too close to the light (and there's no eye response to the increasing amount of light hitting your eyes), and if it's dark, your pupils are maximally dilated. You don't see microwaves, but above about 20 W, the heat is quite noticeable, and a 200 W light is unmistakably warm (those are usually used for heating, though, not IR camera illumination).
                                    – Luaan
                                    20 hours ago


















                                  • "High intensity" is relative, of course; most spotlights used for "night vision cameras" are still just a few watts, which is rather small compared to sunlight. There is a danger mainly because you don't realize you're too close to the light (and there's no eye response to the increasing amount of light hitting your eyes), and if it's dark, your pupils are maximally dilated. You don't see microwaves, but above about 20 W, the heat is quite noticeable, and a 200 W light is unmistakably warm (those are usually used for heating, though, not IR camera illumination).
                                    – Luaan
                                    20 hours ago
















                                  "High intensity" is relative, of course; most spotlights used for "night vision cameras" are still just a few watts, which is rather small compared to sunlight. There is a danger mainly because you don't realize you're too close to the light (and there's no eye response to the increasing amount of light hitting your eyes), and if it's dark, your pupils are maximally dilated. You don't see microwaves, but above about 20 W, the heat is quite noticeable, and a 200 W light is unmistakably warm (those are usually used for heating, though, not IR camera illumination).
                                  – Luaan
                                  20 hours ago




                                  "High intensity" is relative, of course; most spotlights used for "night vision cameras" are still just a few watts, which is rather small compared to sunlight. There is a danger mainly because you don't realize you're too close to the light (and there's no eye response to the increasing amount of light hitting your eyes), and if it's dark, your pupils are maximally dilated. You don't see microwaves, but above about 20 W, the heat is quite noticeable, and a 200 W light is unmistakably warm (those are usually used for heating, though, not IR camera illumination).
                                  – Luaan
                                  20 hours ago











                                  -1














                                  The trick with microwaves is that they use resonance frequencies of water. Some microwaves can be tuned to meat, vegetables or fish, since the resonance frequency can change slightly in composition. Visible light does not resonate with anything in our bodies.



                                  Our generally photo-sensitive skin mostly reacts to UV-Ranges, even in a not yet ionizing spectrum. (The wavelength also determines how deep the light can enter the skin!)



                                  With enough energy, visible light could be dangerous, but the energy needed is far greater without any resonance effects. (think of a child swing)






                                  share|cite|improve this answer








                                  New contributor




                                  Anonymous is a new contributor to this site. Take care in asking for clarification, commenting, and answering.
                                  Check out our Code of Conduct.


















                                  • Welcome to Physics SE! Hope you'll enjoy it here. Great first answer.
                                    – Andrea
                                    yesterday






                                  • 12




                                    Microwaves don't use resonance; it's a common misconception. See explanation e.g. here or anywhere on the Internet.
                                    – user27542
                                    yesterday






                                  • 1




                                    In what is fish water different from meat water or vegetable water? Not to mention that you can heat up butter just fine in a microwave.
                                    – Dmitry Grigoryev
                                    yesterday








                                  • 1




                                    @DmitryGrigoryev -- butter has a lot of water in it. When you put butter in a hot pan, the bubbles are the water boiling off.
                                    – Pete Becker
                                    yesterday








                                  • 1




                                    I think resonance is the wrong word here. I think the more correct phrasing is that most microwave ovens emit at a frequency that corresponds to a peak in the absorption spectrum of water. This answer is right in noting that absorption is another key factor in addition to the energy per photon and the intensity of light.
                                    – Cogitator
                                    yesterday
















                                  -1














                                  The trick with microwaves is that they use resonance frequencies of water. Some microwaves can be tuned to meat, vegetables or fish, since the resonance frequency can change slightly in composition. Visible light does not resonate with anything in our bodies.



                                  Our generally photo-sensitive skin mostly reacts to UV-Ranges, even in a not yet ionizing spectrum. (The wavelength also determines how deep the light can enter the skin!)



                                  With enough energy, visible light could be dangerous, but the energy needed is far greater without any resonance effects. (think of a child swing)






                                  share|cite|improve this answer








                                  New contributor




                                  Anonymous is a new contributor to this site. Take care in asking for clarification, commenting, and answering.
                                  Check out our Code of Conduct.


















                                  • Welcome to Physics SE! Hope you'll enjoy it here. Great first answer.
                                    – Andrea
                                    yesterday






                                  • 12




                                    Microwaves don't use resonance; it's a common misconception. See explanation e.g. here or anywhere on the Internet.
                                    – user27542
                                    yesterday






                                  • 1




                                    In what is fish water different from meat water or vegetable water? Not to mention that you can heat up butter just fine in a microwave.
                                    – Dmitry Grigoryev
                                    yesterday








                                  • 1




                                    @DmitryGrigoryev -- butter has a lot of water in it. When you put butter in a hot pan, the bubbles are the water boiling off.
                                    – Pete Becker
                                    yesterday








                                  • 1




                                    I think resonance is the wrong word here. I think the more correct phrasing is that most microwave ovens emit at a frequency that corresponds to a peak in the absorption spectrum of water. This answer is right in noting that absorption is another key factor in addition to the energy per photon and the intensity of light.
                                    – Cogitator
                                    yesterday














                                  -1












                                  -1








                                  -1






                                  The trick with microwaves is that they use resonance frequencies of water. Some microwaves can be tuned to meat, vegetables or fish, since the resonance frequency can change slightly in composition. Visible light does not resonate with anything in our bodies.



                                  Our generally photo-sensitive skin mostly reacts to UV-Ranges, even in a not yet ionizing spectrum. (The wavelength also determines how deep the light can enter the skin!)



                                  With enough energy, visible light could be dangerous, but the energy needed is far greater without any resonance effects. (think of a child swing)






                                  share|cite|improve this answer








                                  New contributor




                                  Anonymous is a new contributor to this site. Take care in asking for clarification, commenting, and answering.
                                  Check out our Code of Conduct.









                                  The trick with microwaves is that they use resonance frequencies of water. Some microwaves can be tuned to meat, vegetables or fish, since the resonance frequency can change slightly in composition. Visible light does not resonate with anything in our bodies.



                                  Our generally photo-sensitive skin mostly reacts to UV-Ranges, even in a not yet ionizing spectrum. (The wavelength also determines how deep the light can enter the skin!)



                                  With enough energy, visible light could be dangerous, but the energy needed is far greater without any resonance effects. (think of a child swing)







                                  share|cite|improve this answer








                                  New contributor




                                  Anonymous is a new contributor to this site. Take care in asking for clarification, commenting, and answering.
                                  Check out our Code of Conduct.









                                  share|cite|improve this answer



                                  share|cite|improve this answer






                                  New contributor




                                  Anonymous is a new contributor to this site. Take care in asking for clarification, commenting, and answering.
                                  Check out our Code of Conduct.









                                  answered yesterday









                                  AnonymousAnonymous

                                  391




                                  391




                                  New contributor




                                  Anonymous is a new contributor to this site. Take care in asking for clarification, commenting, and answering.
                                  Check out our Code of Conduct.





                                  New contributor





                                  Anonymous is a new contributor to this site. Take care in asking for clarification, commenting, and answering.
                                  Check out our Code of Conduct.






                                  Anonymous is a new contributor to this site. Take care in asking for clarification, commenting, and answering.
                                  Check out our Code of Conduct.












                                  • Welcome to Physics SE! Hope you'll enjoy it here. Great first answer.
                                    – Andrea
                                    yesterday






                                  • 12




                                    Microwaves don't use resonance; it's a common misconception. See explanation e.g. here or anywhere on the Internet.
                                    – user27542
                                    yesterday






                                  • 1




                                    In what is fish water different from meat water or vegetable water? Not to mention that you can heat up butter just fine in a microwave.
                                    – Dmitry Grigoryev
                                    yesterday








                                  • 1




                                    @DmitryGrigoryev -- butter has a lot of water in it. When you put butter in a hot pan, the bubbles are the water boiling off.
                                    – Pete Becker
                                    yesterday








                                  • 1




                                    I think resonance is the wrong word here. I think the more correct phrasing is that most microwave ovens emit at a frequency that corresponds to a peak in the absorption spectrum of water. This answer is right in noting that absorption is another key factor in addition to the energy per photon and the intensity of light.
                                    – Cogitator
                                    yesterday


















                                  • Welcome to Physics SE! Hope you'll enjoy it here. Great first answer.
                                    – Andrea
                                    yesterday






                                  • 12




                                    Microwaves don't use resonance; it's a common misconception. See explanation e.g. here or anywhere on the Internet.
                                    – user27542
                                    yesterday






                                  • 1




                                    In what is fish water different from meat water or vegetable water? Not to mention that you can heat up butter just fine in a microwave.
                                    – Dmitry Grigoryev
                                    yesterday








                                  • 1




                                    @DmitryGrigoryev -- butter has a lot of water in it. When you put butter in a hot pan, the bubbles are the water boiling off.
                                    – Pete Becker
                                    yesterday








                                  • 1




                                    I think resonance is the wrong word here. I think the more correct phrasing is that most microwave ovens emit at a frequency that corresponds to a peak in the absorption spectrum of water. This answer is right in noting that absorption is another key factor in addition to the energy per photon and the intensity of light.
                                    – Cogitator
                                    yesterday
















                                  Welcome to Physics SE! Hope you'll enjoy it here. Great first answer.
                                  – Andrea
                                  yesterday




                                  Welcome to Physics SE! Hope you'll enjoy it here. Great first answer.
                                  – Andrea
                                  yesterday




                                  12




                                  12




                                  Microwaves don't use resonance; it's a common misconception. See explanation e.g. here or anywhere on the Internet.
                                  – user27542
                                  yesterday




                                  Microwaves don't use resonance; it's a common misconception. See explanation e.g. here or anywhere on the Internet.
                                  – user27542
                                  yesterday




                                  1




                                  1




                                  In what is fish water different from meat water or vegetable water? Not to mention that you can heat up butter just fine in a microwave.
                                  – Dmitry Grigoryev
                                  yesterday






                                  In what is fish water different from meat water or vegetable water? Not to mention that you can heat up butter just fine in a microwave.
                                  – Dmitry Grigoryev
                                  yesterday






                                  1




                                  1




                                  @DmitryGrigoryev -- butter has a lot of water in it. When you put butter in a hot pan, the bubbles are the water boiling off.
                                  – Pete Becker
                                  yesterday






                                  @DmitryGrigoryev -- butter has a lot of water in it. When you put butter in a hot pan, the bubbles are the water boiling off.
                                  – Pete Becker
                                  yesterday






                                  1




                                  1




                                  I think resonance is the wrong word here. I think the more correct phrasing is that most microwave ovens emit at a frequency that corresponds to a peak in the absorption spectrum of water. This answer is right in noting that absorption is another key factor in addition to the energy per photon and the intensity of light.
                                  – Cogitator
                                  yesterday




                                  I think resonance is the wrong word here. I think the more correct phrasing is that most microwave ovens emit at a frequency that corresponds to a peak in the absorption spectrum of water. This answer is right in noting that absorption is another key factor in addition to the energy per photon and the intensity of light.
                                  – Cogitator
                                  yesterday











                                  -1














                                  Stand in front of a 2KW spotlight, like used in stage productions.
                                  You will start to feel it on your skin (or simply blind you). As explained elsewhere, microwaves go deeper and a microwave oven is still delivering a KW of energy to your body.






                                  share|cite|improve this answer








                                  New contributor




                                  Reed Shilts is a new contributor to this site. Take care in asking for clarification, commenting, and answering.
                                  Check out our Code of Conduct.























                                    -1














                                    Stand in front of a 2KW spotlight, like used in stage productions.
                                    You will start to feel it on your skin (or simply blind you). As explained elsewhere, microwaves go deeper and a microwave oven is still delivering a KW of energy to your body.






                                    share|cite|improve this answer








                                    New contributor




                                    Reed Shilts is a new contributor to this site. Take care in asking for clarification, commenting, and answering.
                                    Check out our Code of Conduct.





















                                      -1












                                      -1








                                      -1






                                      Stand in front of a 2KW spotlight, like used in stage productions.
                                      You will start to feel it on your skin (or simply blind you). As explained elsewhere, microwaves go deeper and a microwave oven is still delivering a KW of energy to your body.






                                      share|cite|improve this answer








                                      New contributor




                                      Reed Shilts is a new contributor to this site. Take care in asking for clarification, commenting, and answering.
                                      Check out our Code of Conduct.









                                      Stand in front of a 2KW spotlight, like used in stage productions.
                                      You will start to feel it on your skin (or simply blind you). As explained elsewhere, microwaves go deeper and a microwave oven is still delivering a KW of energy to your body.







                                      share|cite|improve this answer








                                      New contributor




                                      Reed Shilts is a new contributor to this site. Take care in asking for clarification, commenting, and answering.
                                      Check out our Code of Conduct.









                                      share|cite|improve this answer



                                      share|cite|improve this answer






                                      New contributor




                                      Reed Shilts is a new contributor to this site. Take care in asking for clarification, commenting, and answering.
                                      Check out our Code of Conduct.









                                      answered 12 hours ago









                                      Reed ShiltsReed Shilts

                                      991




                                      991




                                      New contributor




                                      Reed Shilts is a new contributor to this site. Take care in asking for clarification, commenting, and answering.
                                      Check out our Code of Conduct.





                                      New contributor





                                      Reed Shilts is a new contributor to this site. Take care in asking for clarification, commenting, and answering.
                                      Check out our Code of Conduct.






                                      Reed Shilts is a new contributor to this site. Take care in asking for clarification, commenting, and answering.
                                      Check out our Code of Conduct.

















                                          protected by rob 5 hours ago



                                          Thank you for your interest in this question.
                                          Because it has attracted low-quality or spam answers that had to be removed, posting an answer now requires 10 reputation on this site (the association bonus does not count).



                                          Would you like to answer one of these unanswered questions instead?



                                          Popular posts from this blog

                                          Liste der Baudenkmale in Friedland (Mecklenburg)

                                          Single-Malt-Whisky

                                          Czorneboh